You are on page 1of 79

www.insightsonindia.

com
www.insightsias.com
TEST – 5 Solutions

INSIGHTS PRELIMS MOCK TESTS 2016

TEST – 5

SOLUTIONS

1. Solution: a)

Learning: The USO is situated on an island in the middle of the Lake


Fatehsagar of Udaipur, Rajasthan, India. The sky conditions at Udaipur
are quite favourable for solar observations. The large water body
surrounding the telescopes decreases the amount of heating of the
surface layers. This decreases the turbulence in the air mass and thereby
improves the image quality and seeing.

The main objective of obtaining the high spatial and temporal resolution
observations of solar photospheric and chromospheric activity is to
understand the various dynamic phenomena occurring on the surface of
the Sun.

Q Source: http://www.isro.gov.in/about-isro/multi-application-solar-
telescope-operationalised-udaipur-solar-observatory

2. Solution: d)
www.insightsonindia.com
www.insightsias.com
TEST – 5 Solutions

Learning: The New International Economic Order (NIEO) was a set of


proposals put forward during the 1970s by some developing countries
through the United Nations Conference on Trade and Development to
promote their interests by improving their terms of trade, increasing
development assistance, developed-country tariff reductions, and other
means.

It was meant to be a revision of the international economic system in


favour of Third World countries, replacing the Bretton Woods system,
which had benefited the leading states that had created it – especially the
United States.

Q Source: Chapter 4: Page 100: 10th NCERT World History

3. Solution: d)

Learning: In a world first event, in October 2013, leaders in the


governments of all 12 snow leopard range countries came together at the
Global Snow Leopard Forum initiated by the President of Kyrgyz
Republic, Mr Almazbek Atambayev.

At that meeting in the Kyrgyz Republic’s capital, Bishkek, the 12


countries agreed that the snow leopard and the high mountain habitat it
lives in, is too precious to let disappear.

To help spread the word amongst the people, government authorities


and conservation groups in each country, designated 2015 as
International Year of the Snow Leopard in Afghanistan, Bhutan, China,
India, Kazakhstan, Kyrgyz Republic, Mongolia, Nepal, Pakistan, Russia,
Tajikistan and Uzbekistan.

Q Source: http://www.moef.nic.in/public-information/protected-area-
network

4. Solution: d)

Justification: USSR was a nation-state. Authority was centralized in it.


So, (a) is wrong.
www.insightsonindia.com
www.insightsias.com
TEST – 5 Solutions

A nation-state is a mix of several ethnicities, a state need NOT be. So, (b)
is wrong.

Both nation-states (e.g. USA, Canada) and states (e.g. India) can have
constitution. So, (c) is wrong.

Learning: A nation refers only to a socio-cultural entity, a union of


people sharing who can identify culturally and linguistically. This
concept does not necessarily consider formal political unions.

A state refers to a legal/political entity that is comprised of the following:


a) a permanent population; b) a defined territory; c) a government ; and
d) the capacity to enter into relations with other states.

Combination of both is known as a nation-state.

Q Source: 10th NCERT World History

5. Solution: a)

Justification: The Biological Diversity Act 2002 covers conservation,


use of biological resources and associated knowledge occurring in India
for commercial or research purposes or for the purposes of bio-survey
and bio-utilisation.

It establishes, NBA, SSBs and local level bodies.

Statement 2 is done by local level Biodiversity Management Committees


(BMCs) constituted under the Biological Diversity Act (2002).

You read more about all the functions of NBA and the Act here.

Q Source: http://nbaindia.org/content/22/2/1/aboutnba.html

6. Solution: a)

Justification & Learning: at the UN monetary and financial


Conference held in 1944, at Bretton Woods, USA, it was decided to
www.insightsonindia.com
www.insightsias.com
TEST – 5 Solutions

establish IMF and International Bank for reconstruction and


Development (World Bank).

IMF would deal with external surpluses and deficits of member nations.
IBRD would setup to finance post-war reconstruction. So, 1 and 2 is
correct.

The goal of full employment is dealt with national governments. So, 3 is


wrong.

Q Source: Chapter 4: Page 100: 10th NCERT World History

7. Solution: d)

Justification & Learning: The IWT advantages are several and


disadvantages are few. Among all the transport modes, it is the least
capital-intensive, is environment-friendly, can supplement rail and road
transport, help in the decongestion of roads, is best suited to carry over-
dimensional cargo (ODC), requires minimum. So, 2 is wrong.

Transportation costs are lesser than roadways. So, 3 is wrong.

Both IWT and coastal shipping are highly fuel efficient when compared
to roadways. So, 1 is wrong.

Rivers, canals and lakes can be used as IWT.

For the Twelfth Five Year Plan (2012-2017), IWT has been allocated
R10,500 crore, substantially more than the previous Plan periods. A
World Bank study on IWT has shown that “in many instances, these
advantages can be gained with little or no improvement to existing
waterways. In others, a modest level of complementary investment can
significantly increase usability”.

Q Source: Yojana November 2015 – Inside Cover Page

8. Solution: d)

Justification: This is one more question based on the concept of


weight and gravity.
www.insightsonindia.com
www.insightsias.com
TEST – 5 Solutions

The moon orbits the Earth once every 27.322 days. It also takes
approximately 27 days for the moon to rotate once on its axis. The earth
(despite being larger) takes only 24 hours to rotate on its axis. So, clearly
earth rotates faster than moon.

Moon’s gravity has no relation with its rotation. But, yes, the weight of a
body on moon will be affected by Moon’s rotation.

This is because, your weight also depends on the centrifugal force


exerted due to the rotation of the body.

W = G – Centrifugal force

Q Source: Improvisation: Chapter 10: Page 137: 9th Science NCERT

9. Solution: d)

Justification: The rate at which the RBI lends money to commercial


banks is called repo rate. It is an instrument of monetary policy.
Whenever banks have any shortage of funds they can borrow from the
RBI.

Reverse Repo rate is the rate at which the RBI borrows money from
commercial banks. The difference between both rates is always
maintained at 100 basis points (or 1%). So, if repo rate changes, it
changes reverse repo also. Hence, 1 is correct and 2 and 3 are wrong.
CRR and SLR are fixed separately.

Q Source:
https://www.rbi.org.in/Scripts/BS_PressReleaseDisplay.aspx?prid=355
95

10. Solution: b)

Justification: President of India is the chief commander of the Indian


armed forces. Any decision to conclude peace or war is finally approved
by him.
www.insightsonindia.com
www.insightsias.com
TEST – 5 Solutions

However, after his approval, the Parliament must also assent to it within
a certain period of time.

So, (b) is correct. Others only play an advisory role. Final decision taken
by cabinet is forwarded to the President.

Learning: The question was made in backdrop of German cabinet


approving a mandate offering military assistance to fight against the
Islamic State (IS) militant group in Syria.

Under this mandate Germany could deploy up to 1,200 military


personnel to support the international alliance but they won’t actively
engage in combat missions.

Q Source: http://www.business-standard.com/article/news-
ians/german-cabinet-clears-anti-terror-military-aid-
115120100945_1.html

11. Solution: b)

Justification: In railways, only Central government invests. Some


services may be outsourced to private parties (e.g. catering). So, 1 is
wrong.

In civil aviation, airports are usually built under the PPP model, where
both Central government and private parties invest jointly and share the
revenues and risks. So, 2 is correct.

Learning: In India, railways are under-invested. It accounts for only


5.5% of total plan outlay vis-a-vis 11% in other transport sectors.

Network expansion in Railways has lagged behind capacity addition in


the domestic roads sector. So, significant freight share has gone to the
roadways (57% of total), instead of more efficient railways.

So, there is a strong case for channeling resources to such transport


infrastructure in India given the widely known spillover effects of
transport networks to link markets, reduce a variety of costs, boost
www.insightsonindia.com
www.insightsias.com
TEST – 5 Solutions

agglomeration economies, and improve the competitiveness of the


economy, especially manufacturing which tends to be logistics-intensive.

Q Source: Yojana November 2015 – Page 29-30

12. Solution: b)

Justification: For e.g. recently the President assented to the Labour


Laws (Gujarat Amendment) Bill, 2015. It has provisions relating to

• Ban strikes: Empowers state government to ban strikes in public


utility services
• Dispute Settlement: Add provision for out of court settlement
between the labourers and management by paying certain fee to
the government.
• Redefine contractors: Amends the definition of contractor to
include outsourcing agencies, which in some cases is government
itself.
• Payment of wages: Mandatory to be paid by cheque instead of cash
for establishments employing 20 persons or more.
• Empower employers to change nature of jobs of the employees
without prior notice.

Hence, all options except 4 are correct. This is because minimum


qualifications are not generally regulated by laws except in some cases
such as medical schools etc. Instead, specialized bodies (for e.g. ICAI for
CA) regulate minimum qualifications.

Q Source: http://www.thehindu.com/news/national/gujarats-
controversial-labour-laws-bill-gets-presidential-
assent/article7938265.ece

13. Solution: d)
www.insightsonindia.com
www.insightsias.com
TEST – 5 Solutions

Justification: They can be used in Himalayan Regions, glaciers and


even sea ports. So, 1 is incorrect.

Bio-toilets are generally maintenance free and have a life of upto 50


years at once. This is why they are being installed at large scale in
railways. So, 2 is also incorrect.

Learning:

Q Source: Improvisation: Yojana November 2015

14. Solution: c)

Justification & Learning: Performed in the state of Assam on Bihu


festival, it is characterized by brisk dance steps,and rapid hand
movement. Dancers (both genders) wear traditionally colorful Assamese
clothing. So, 1 is correct and 2 is incorrect.

The dance takes several forms among the different northeast Indian
groups, e.g. "Deori Bihu dance", "Mising Bihu dance" etc. However, the
www.insightsonindia.com
www.insightsias.com
TEST – 5 Solutions

underlying goal of the dance remains the same: to express the desire to
feel both pain and happiness.

Bihu generally marks the end of the harvest season. So, 3 is wrong.

Q Source: Improvisation: Chapter 1: Page 3 (picture): 9th Science


NCERT

15. Solution: c)

Justification: The PMKKKY fund is obtained after imposing royalty


payments on the miners. So, the funds will not be used to subsidize
mining company or units. It will be used exclusively for the welfare of the
citizens affected by mining. So, statement 2 is incorrect.

Welfare would involve overall development, i.e. watershed development,


reducing pollution, improving health indicators, education etc. So, 1 and
3 are correct.

Learning: Areas of focus:

• High priority areas like drinking water supply, health care,


sanitation, education, skill development, women and child care,
welfare of aged and disabled people, skill development and
environment conservation will get at least 60 % share of the
funds.
• For creating a supportive and conducive living environment,
balance funds will be spent on making roads, bridges, railways,
waterways projects, irrigation and alternative energy sources.

About District Mineral Foundations (DMFs):

• The Mines and Minerals (Development & Regulation)


Amendment Act, 2015, mandated the setting up of District
Mineral Foundations (DMFs) in all districts in the country
affected by mining related operations.

Q Source: Yojana November 2015 – Page 62


www.insightsonindia.com
www.insightsias.com
TEST – 5 Solutions

16. Solution: c)

Justification: Mammals are warm-blooded animals with four-


chambered hearts. They have mammary glands for the production of
milk to nourish their young. Their skin has hairs as well as sweat and oil
glands.

Most mammals familiar to us produce live young ones. However, a few of


them, like the platypus and the echidna lay eggs, and some, like
kangaroos give birth to very poorly developed young ones.

Whales, dolphins, porpoises, seals, walruses, and many other marine


animals are mammals, not fish. Apart from the options mentioned
above, they have fur (although it is very sparse on their body) and they
provide milk for their young.

Q Source: Chapter 7: Page 94: 9th Science NCERT

17. Solution: a)

Learning: NATO member Turkey recently in November 2015 had shot


down Russian Sukhoi 24 (Su-24) warplane escalating crisis in bilateral
relations of both countries.

Turkey has claimed that the Russian jet was shot down after it had
crossed into its airspace and ignored several warnings to change course.
However, Russia has furiously denied the claims and demanded an
apology and economic compensation from Turkey.

Russian economic sanctions covers Ban on imports from Turkey, work of


Turkish companies in Russia and any Turkish nationals working for
Russian companies. It also covers end to charter flights between the
countries and urges Russian tour operators to refrain from selling tourist
packages to Turkey.

Q Source: http://www.reuters.com/article/2015/12/01/us-mideast-
crisis-russia-turkey-sanction-idUSKBN0TK4SD20151201
www.insightsonindia.com
www.insightsias.com
TEST – 5 Solutions

18. Solution: c)

Learning: Here are the criteria mentioned on the ECI website.

A political party shall be treated as a recognised political party in a State,


if and only if either the conditions specified in Clause (A) are, or the
condition specified in Clause (B) is, fulfilled by that party and not
otherwise, that is to say-

(A) that such party –

has been engaged in political activity for a continuous period of five


years; and

has, at the last general election in that State to the House of the People,
or, as the case may be, to the Legislative Assembly of the State, returned-

either (i) at least one member to the House of the People for every
twenty-five members of that House or any fraction of that number from
that State;

or (ii) at least one member to the Legislative Assembly of that State for
every thirty members of that Assembly or any fraction of that number;

To read clause (B) visit


http://eci.nic.in/eci_main1/RegisterationPoliticalParties.aspx

Q Source: Chapter 6: Page 11: 10th NCERT Polity

19. Solution: d)

Justification & Learning: The Association for Democratic Reforms


(ADR) is a non-partisan, non-governmental organization which works in
the area of electoral and political reforms.

Along with National Election Watch (NEW), which is a conglomeration


of over 1200 organizations across the country, ADR aims at bringing
www.insightsonindia.com
www.insightsias.com
TEST – 5 Solutions

transparency and accountability in Indian politics and reducing the


influence of money and muscle power in elections.

Function & Research Areas

• Election Watch or Analysis of Affidavits


• Political Party Watch (PPW):
• legal advocacy for Political and Electoral Reforms
• Dissemination of information on elected representatives

Q Source: Improvisation: Chapter 5: 10th NCERT Polity

20. Solution: d)

Justification: The actual reason is atmospheric refraction, which was


covered in the last Test.

A mirage has nothing to do with the flattening of the Sun’s discs. So, 1 is
incorrect.

A Halo is an optical phenomenon produced by light interacting with ice


crystals suspended in the atmosphere, resulting in a wide variety of
colored or white rings, arcs and spots in the sky. It’s not the reason for
the flattening of the disc. So, 2 is wrong.

Cloud iridescence is the occurrence of colors in a cloud similar to those


seen in oil films on puddles, and is similar to irisation. So, 3 is also
incorrect.

Q Source: Xth NCERT

21. Solution: d)

Justification: Here is the table comparing growth rates in the recent


past. There is no evidence that democracies grow faster than
dictatorships. In fact, the reverse has been found to be true. So, 1 is
wrong.
www.insightsonindia.com
www.insightsias.com
TEST – 5 Solutions

Non-democratic nations like Saudi Arabia, Syria are all members of UN.
So, even statement 2 is wrong.

Q Source: Chapter 7: page 94: 10th NCERT Polity

22. Solution: d)

Justification: Science has not learnt to even understand dark energy.


The question of harnessing is distant. So, (a) is incorrect.

Microwaves are a part of the electromagnetic spectrum, which are used


for communication, not generating energy. So, (c) is incorrect.

Gravitational waves are primordial waves of the Universe. So, (c) is


incorrect.

Learning: Solar cells, batteries and nuclear cells are some of the
sources of energy for artificial satellites and space probes. Solar cells also
have other uses. Radio or wireless transmission systems or TV relay
stations in remote location use solar cell panels.

Traffic signals are also fitted with solar cells. The solar cell panels are
mounted on specially designed inclined roof tops so that more solar
energy is incident over it. The domestic use of solar cells is, however,
limited due to its high cost.

Q Source: Xth NCERT

23. Solution: c)
www.insightsonindia.com
www.insightsias.com
TEST – 5 Solutions

Justification: India has hosted three IBSA (founded in 2003) summits,


all in New Delhin in 2008, 2013 and 2015 (planned). So, 2 is wrong.

The main objective of the IBSA Fund (established in 2004) is to benefit


other developing countries, particularly Least Developed Countries
(LDCs) and Post Conflict Reconstruction and Development (PCRD)
countries around the world in partnership with the UN. So, 4 is correct.

Learning: This is a good article that compares the role of BRICS and
IBSA.

We feel a question might come in this year UPSC Mains, or next year
Prelims.

Q Source: http://www.thehindu.com/opinion/lead/the-bricks-to-
rebuild-ibsa/article7147509.ece

24. Solution: b)

Justification: Tuberculosis (TB) is caused by bacteria (Mycobacterium


tuberculosis), not coliform. So, 1 is incorrect.

Natural gas deposits occur generally with oil. It has nothing to do with
coliform bacteria. So, 3 is incorrect.

Coliform bacteria are a commonly used bacterial indicator of sanitary


quality of foods and water (e.g. in Ganges water). So, 2 is correct.

Learning: Coliforms can be found in the aquatic environment, in soil


and on vegetation; they are universally present in large numbers in the
feces of warm-blooded animals. While coliforms themselves are not
normally causes of serious illness, they are easy to culture, and their
presence is used to indicate that other pathogenic organisms of fecal
origin may be present.

E. coli), a rod-shaped member of the coliform group. Most strains of E.


coli are harmless, but some can cause serious illness in humans.
Infection symptoms and signs include bloody diarrhea, stomach cramps,
vomiting and occasionally, fever.

Q Source: Xth NCERT


www.insightsonindia.com
www.insightsias.com
TEST – 5 Solutions

25. Solution: d)

Justification: The constitution of political parties defines their


composition, governance structures, mandate, vision, mission etc. It
serves as an inner check on the party members and cannot be violated. It
need not be approved by anybody.

However, if the manifestos issued by the political parties contain


something objectionable, an appeal can be filed to the ECI. OR if a
registered political party has an objectionable constitution (e.g. seditious
content), it can be banned (e.g. Maoist parties) by the government and
ECI.

Q Source: Chapter 6: page 75: 10th NCERT Polity

26. Solution: b)

Justification: Crops production can go down due to biotic (diseases,


insects and nematodes) and abiotic (drought, salinity, water logging,
heat, cold and frost) stresses under different situations. Varieties
resistant to these stresses can improve crop production. So, 2 and 3 are
correct.

Agronomic characteristics are tallness, dwarfness etc of plants. Tallness


and profuse branching are desirable characters for fodder crops.
Dwarfness is desired in cereals. But, these are not part of biotic and
abiotic stresses. So, 1 is incorrect.

Learning: Varieties or strains of crops can be selected by breeding for


various useful characteristics such as disease resistance, response to
fertilisers, product quality and high yields.

One way of incorporating desirable characters into crop varieties is by


hybridisation. Hybridisation refers to crossing between genetically
dissimilar plants. This crossing may be intervarietal (between different
varieties), interspecific (between two different species of the same genus)
or intergeneric (between different genera).
www.insightsonindia.com
www.insightsias.com
TEST – 5 Solutions

Another way of improving the crop is by introducing a gene that would


provide the desired characteristic. This results in genetically modified
crops.

Q Source: Chapter 15: Page 204: 9th Science NCERT

27. Solution: b)

Justification: SEBI already monitors speculation in stock market. So,


(a) is not appropriate answer, given (b).

Fining retailers for hoarding comes under the jurisdiction of local


governments. So, (c) is wrong.

SEBI cannot regulate any mergers and acquisitions. It falls under the
domain of other regulatory authorities like CCI. So, (d) is also wrong.

Learning: Under the Forward Contracts (Regulation) Act, 1952, which


regulates commodity trading in India, a forward contract is a contract for
the actual delivery of goods. On the other hand, a futures contract is one
where the buyer can settle the contract in cash as well.

With SEBI-FMC merger, SEBI is set to take over regulation of


commodities markets as well. However, it is not comfortable with
forward contracts in commodity exchanges. · SEBI’s concern stems from
two facts:

• One, unlike futures contracts, forward contracts are not


standardised;
• Two, there’s greater counterparty risk associated with forward
contracts.

You can also read the article below to understand it better.

Q Source: http://indianexpress.com/article/explained/merger-of-sebi-
and-fmc-what-it-means-for-market-players/

28. Solution: a)
www.insightsonindia.com
www.insightsias.com
TEST – 5 Solutions

Justification: Iron content in the crop will remain the same before and
after drought. So, (b) can’t be the answer.

Light soils can be as easily irrigated, and in fact more frequently, as


dense soils. So, (c) is incorrect.

They don’t change mineral nutrient composition on drying. In severe


drought, organic matter may however dry up. So, (d) is also incorrect.

Learning: Sandy soils have high proportion of sand and little clay. Also
known as light soils, these soils drain quickly after rain or watering, are
easy to cultivate and work. They warm up more quickly in spring than
clay soils. But on the downside, they dry out quickly and are low in plant
nutrients, which are quickly washed out by rain. Sandy soils are often
very acidic.

Q Source: Chapter 15: Page 207: 9th Science NCERT

29. Solution: d)

Learning: The Federico Fellini Gold Medal was created by UNESCO in


1995 to pay tribute to major figures in the film industry, and to notable
efforts to boost the art of film.

International Film Festival India (IFFI) 2015 in collaboration with the


International Council for Film, Television and Audiovisual
Communication (ICFT), Paris will present a special ICFT prize consisting
of the UNESCO Fellini Medal, awarded to a film, which reflects the
ideals promoted by UNESCO.

Five Indian films have been shortlisted from the official selection of IFFI
following the recommendations of the Film Preview Committee
constituted by the Directorate of the Film Festivals. This award will be
given at IFFI for the first time.

Q Source: http://pib.nic.in/newsite/mbErel.aspx?relid=131991

http://timesofindia.indiatimes.com/city/mumbai/UnescoFellini-award-
to-be-presented-at-IFFI-this-year/articleshow/49952872.cms
www.insightsonindia.com
www.insightsias.com
TEST – 5 Solutions

30. Solution: d)

Learning: In case the ULBs do not have adequate capacity to handle


projects, the State Government may recommend in State Action Plan
(SAAP), upon a Resolution passed by the ULB, for the execution of the
projects by specialized parastatal agencies of the State or Central
Governments.

Public Private Partnerships (PPP), which should be the preferred


execution model. The PPP should include appropriate Service Level
Agreements (SLAs) with strong citizen feedback built into it. This will
lead to the People Public Private Partnership (PPPP) model.

To know the focus areas of the scheme, you can refer to


http://amrut.gov.in/writereaddata/AMRUT%20Guidelines%20.pdf

Q Source: http://www.thehindu.com/news/national/kerala/amrut-to-
roll-out-on-a-smaller-scale/article7868200.ece

31. Solution: d)

Justification & Learning: Plants generally take up nitrates and


nitrites and convert them into amino acids which are used to make
proteins.

These proteins and other complex compounds are subsequently


consumed by animals.

Once the animal or the plant dies, other bacteria in the soil convert the
various compounds of nitrogen back into nitrates and nitrites. A
different type of bacteria converts the nitrates and nitrites into elemental
nitrogen.

Thus, there is a nitrogen-cycle in nature in which nitrogen passes from


its elemental form in the atmosphere into simple molecules in the soil
and water, which get converted to more complex molecules in living
beings and back again to the simple nitrogen molecule in the
atmosphere.
www.insightsonindia.com
www.insightsias.com
TEST – 5 Solutions

Q Source: Chapter 14: Page 198: 9th Science NCERT

32. Solution: a)

Justification & Learning: The oxygen we find in our atmosphere is


made up of two oxygen atoms (O2). Because of its chemical formulation,
a single atom of oxygen (O) is unstable. That is, it wants to combine with
something else. That is why oxygen is almost always found in pairs, in its
(diatomic) form, where it is more stable. O3 is less stable than O2,
because it wants to return to the diatomic state by giving up an oxygen
atom.

When enough ozone molecules are present, it forms a pale blue gas. It is
an unstable molecule that readily combines with other atoms. So, (a) is
correct.

Ozone has the same chemical structure whether it is found in the


stratosphere or the troposphere, and so has the same warming potential.
Option (b) is thus wrong.

Ultraviolet rays do not cause warming, infrared does. So, (c) is also
wrong.

Q Source: Chapter 14: Page 200: 9th Science NCERT

33. Solution: c)

Justification: The concept of APA is related with that of transfer


pricing. It has been covered in the last test. We repeat it here again.

In the past, several Multi-National Companies (MNCs) operating in


India were often accused of misusing the transfer pricing system. MNCs
like Vodafone, Shell, WNS and Nokia were mainly involved in this issue
by misusing the system for transferring profits to their subsidiaries in
countries having low tax rates.
www.insightsonindia.com
www.insightsias.com
TEST – 5 Solutions

By signing APAs, Government is seeking to foster an environment of co-


operation in matters of taxation through predictability of laws and
reducing litigation cases in future. It is considered as a major push
towards providing certainty to foreign investors in the arena of transfer
pricing.

Q Source: http://pib.nic.in/newsite/PrintRelease.aspx?relid=131961

34. Solution: c)

Justification: Speed of sound depends on the density of the medium.


Here the speed of sound is highest in sea water, followed by distilled
water and hydrogen.

So, your friend will hear you earliest through Sea water medium and
latest in Hydrogen filled medium. So, (c) is correct.

Learning: The speed of sound in a medium depends also on


temperature and pressure of the medium. The speed of sound decreases
when we go from solid to gaseous state. In any medium as we increase
the temperature the speed of sound increases.

For example, the speed of sound in air is 331 m/s at 0 degrees Celsius
and 344 m/s at 22 degrees Celsius.

Q Source: Chapter 12: Page 167: 9th Science NCERT

35. Solution: d)

Justification: Bharat stage (BS) norms are emission standards decided


by the government to regulate the amount of air pollutants from
vehicles.

They are not imposed on any industry as such. So, all statements are
wrong.
www.insightsonindia.com
www.insightsias.com
TEST – 5 Solutions

Learning: They were adopted in line with European regulation


standard ‘Euro norms’ in 2000. So far, 4 states of Bharat stage (BS)
norms have been issued by government. In each stage certain limit is
specified on the released pollutants and in the succeeding higher stages,
the BS norms reduces the limit of pollutant emission

Union Government has recently decided to advance the date for


implementation of Bharat Stage (BS) V and BS VI emission norms for
four-wheelers in order to reduce vehicular air pollution from road
transport sector.

Q Source: http://www.business-standard.com/article/pti-stories/govt-
advances-roll-out-of-bs-v-and-bs-vi-norms-for-4-wheelers-
115112800399_1.html

36. Solution: c)

Justification: It started as a fight against French colonial capture. So,


(b) is a factor.

Japan has acquired some territories of Indo-China by force, not North


Korea. In fact, the latter was a supporter of communist forces in
Vietnam. So, (c) is incorrect.

USA’s cold war policy of containment of communism was a major factor


in fuelling the Vietnam War. So, (a) is a factor.

Ousting and assassination of Ngô Đình Diệm also insued political


instability in Vietnam, destabilized the region and fuelled the war
further. So, (d) was a factor.

Q Source: 10th NCERT World History

37. Solution: d)
www.insightsonindia.com
www.insightsias.com
TEST – 5 Solutions

Justification: There is no provision of subsidy as such.

GIAN Scheme aims at improving the quality of higher education in the


country through international collaboration. Facilitate participation of
high quality international academicians for delivering short-term
courses and programs in Indian institutions.

Initially 500 international faculties will be engaged in conducting


courses and later in subsequent years 1000 faculties would be engaged
under GIAN throughout India.

So, both 1 and 2 are wrong. They do not come under the ambit of GIAN
scheme.

Q Source: http://news.niticentral.com/2015/11/30/smriti-irani-
launches-the-global-initiative-of-academic-networks-gian/

38. Solution: d)

Justification: Gravity acts only on particles having some mass. So, 2 is


incorrect.

Tides are caused due to the gravitational forces of the sun and moon. So,
1 is correct.

Gravity is the dominant force at the macroscopic scale, which is the


cause of the formation, shape, and trajectory (orbit) of astronomical
bodies, including those of asteroids, comets, planets, stars, and galaxies.
So, 3 is correct.

Learning: You can refer to Test-3 where the concept of gravity was
covered.

Q Source: Chapter 10: Page 131: 9th Science NCERT

39. Solution: b)
www.insightsonindia.com
www.insightsias.com
TEST – 5 Solutions

Justification: Local bodies like PRIs, municipalities fall under the


State List. It contains subjects of State and local importance such as
police, trade, commerce, agriculture and irrigation.

The State Governments alone can make laws relating to the subjects
mentioned in the State List.

However, this does not stop the Parliament to amend the constitution
with the consent of majority of state government with regard to local
bodies, as it did in the 73rd and 74th amendment.

Q Source: Chapter 2: Page 17: 10th NCERT Polity

40. Solution: d)

Justification: Take the example of India.

Legislative power is divided by 7th schedule of constitution – Union list,


State list, Concurrent List and Residuary list.

Administrative powers are divided by several articles such as A256, 355


etc.

Financial powers are divided in A245-293 (e.g. A280 Finance


Commission).

Diplomatic powers are divided in the 7th schedule itself. Only Central
government has the power to enter into international conventions etc.

The same goes for military, which is shared exclusively with the Centre.

Q Source: Chapter 2: 10th NCERT Polity

41. Solution: a)

Justification: Option (c) was popular even prior to Adam Smith gave
his laissez fairez theory.
www.insightsonindia.com
www.insightsias.com
TEST – 5 Solutions

The role of agriculture in industrial development had been recognized


only recently. So, (b) is wrong.

Centrally planned economic system was the socialistic prescription of the


Soviet Era. Keynes did not advocate central planning; he instead
advocated appropriate fiscal and monetary policies by government which
could stabilize an economy in case of a shock.

He argued that free market system does not always lead to full
employment, economic stability and welfare of citizens. So, (d) is
incorrect and (a) is correct.

Q Source: Chapter 4: Page 97: 10th NCERT World History

42. Solution: a)

Justification: NHAI was created through the promulgation of the


National Highways Authority of India Act, 1988. In February 1995, the
Authority was formally made an autonomous body.

It is also a nodal agency of the Ministry of Road Transport and


Highways. So, 1 and 2 both are correct.

The Cabinet Committee on Economic Affairs (CCEA) approves large PPP


projects in highways. NHAI implements them. So, 3 is incorrect.

Q Source: Improvisation: Yojana November 2015 – Page 27

43. Solution: d)

Learning: The second Anti-Submarine Warfare (ASW) Corvette INS


Kadmatt has been formally handed over to Indian Navy. It is an
indigenous anti-submarine warship.

INS Kadmatt is super-sophisticated frontline ship having following


features.
www.insightsonindia.com
www.insightsias.com
TEST – 5 Solutions

• It has been equipped with total atmospheric control ventilation


system which makes warship fully capable to fight in nuclear,
biological and chemical warfare environments. Ship’s main role:
• It will protect nation’s maritime interests against possible
submarine attack. Provide potent platform for neutralising the
enemy submarines using deployed weapons like torpedoes, rocket
launchers and helicopter.

Q Source:
http://economictimes.indiatimes.com/news/defence/indian-navy-gets-
second-indigenous-anti-submarine-warship-
kadmatt/articleshow/49936650.cms

44. Solution: a)

Justification: Truman doctrine is the USA cold war policy of


communism containment. It is not related to the World War I. So, 3 is
incorrect.

Race for colonies and European domination took the following forms-
competition to control declining Ottoman Empire e.g. Secret pacts like
Sykes Picot Agreement 1916 entered b/w britain and Russia for dividing
Ottoman Empire etc. So, 2 is correct.

Pan Slav movement - All slave countries wanted to Unite - like


unification of Germany, Italy, but Austria resisted it. It was a major
factor behind WW-I. So, 1 is correct.

Q Source: 10th NCERT World History & Contemporary events (2014


marked 100 years of WW-I)

45. Solution: d)

Justification & Learning: Statement 4: Fungi colonize plant roots,


establish chemical communication with the plant

This chemical communication changes plant gene expression and


provides a range of benefits to plant performance.
www.insightsonindia.com
www.insightsias.com
TEST – 5 Solutions

They induce systemic resistance to disease, they increase substantially to


resistance to abiotic stress, and they improve the efficiency of nitrogen
use. They also frequently increase plant growth and deep rooting.

All of these processes require energy, and these fungi induce improved
photosynthetic efficiency in plants. Hence, statement 4 is correct.

Statement 1: Certain mushrooms enjoy usage as therapeutics in folk


medicines, such as Traditional Chinese medicine. So, 1 is correct.

Statement 2: In agriculture, fungi may be useful if they actively compete


for nutrients and space with pathogenic microorganisms such as bacteria
or other fungi. For example, certain fungi may be used to eliminate or
suppress the growth of harmful plant pathogens, such as insects, mites,
weeds, nematodes. So, 2 is correct.

Statement 3: Certain fungi, in particular "white rot" fungi, can degrade


insecticides, herbicides, pentachlorophenol, creosote, coal tars, and
heavy fuels and turn them into carbon dioxide, water, and basic
elements. Fungi have been shown to biomineralize uranium oxides,
suggesting they may have application in the bioremediation of
radioactively polluted sites. So, 3 is correct.

Q Source: Chapter 7: Page 87: 9th Science NCERT

46. Solution: d)

Justification: JNNURM was subsumed under other urban


development schemes of Smart Cities, AMRUT etc. So, 1 is incorrect.

Slum development scheme (Rajiv Gandhi) is not a part of Swachha


Bharat Abhiyaan (see objectives below). So, 2 is incorrect.

NSS is about voluntary service. People participate for any social cause
under NSS. It has not been subsumed. The scheme remains intact. So, 3
is incorrect.

Learning: This campaign aims to accomplish the vision of a 'Clean


India' by 2 October 2019, the 150th birthday of Mahatma Gandhi.
Specific objectives are:
www.insightsonindia.com
www.insightsias.com
TEST – 5 Solutions

• Eliminate open defecation by constructing toilets for


households, communities

• Eradicate manual scavenging

• Introduce modern and scientific municipal solid waste


management practices

• Enable private sector participation in the sanitation sector

• Change people’s attitudes to sanitation and create awareness

• The program plans to construct 12 crore toilets in rural India by


October 2019, at a projected cost of ₹1.96 lakh crore

Q Source: Yojana November 2015 – Page 66

47. Solution: d)

Justification: In order to carry on photosynthesis, green plants need a


supply of carbon dioxide and a means of disposing of oxygen. Oxygen
and glucose is produced in photosynthesis, when CO2 and water
combine in the presence of sunlight and chlorophyll. So, only 2 is
correct.

Learning: In order to carry on cellular respiration, plant cells need


oxygen and a means of disposing of carbon dioxide.

Q Source: Chapter 6: Page 72: 9th Science NCERT

48. Solution: c)

Justification & Learning: A captive power plant is a facility that is


dedicated to providing a localised source of power to an energy user.

These are typically industrial facilities or large offices. The plants may
operate in grid parallel mode with the ability to export surplus power to
the local electricity distribution network.

Alternatively they may have the ability to operate in island mode; i.e.
independently of the local electricity distribution system.
www.insightsonindia.com
www.insightsias.com
TEST – 5 Solutions

Since the announcement of private power policy of the Government of


India in 1991, a number of proposals, including a large number of
proposals from foreign promoters, have been received through
Independent Power Producer (IPP) route.

Q Source: Improvisation: Yojana November 2015 – Page 59

49. Solution: d)

Justification: DPSP: Article 48A : Protection and improvement of


environment and safeguarding of forests and wild life

FR: Article 21A: Right to Clean environment

73rd amendment: Land improvement, implementation of land reforms,


land consolidation and soil conservation; Fuel and fodder; Social forestry
and farm forestry; Maintenance of community assets. Etc can be
devolved to PRIs.

74th amendment: Public health, sanitation, conservancy and solid waste


management; Urban forestry, protection of the environment and
promotion of ecological aspects etc. can be devolved to ULBs.

Q Source: HLC report Nov 2014 MoEF

http://envfor.nic.in/sites/default/files/press-
releases/Final_Report_of_HLC.pdf

50. Solution: d)

Justification: China was not a co-founder of G77. So, (a) is incorrect.

NAM was founded by a group of five nations India, Egypt, Indonesia,


Ghana and Yogoslavia. So, (b) is incorrect.

There are Chapters of the Group of 77 in Rome (FAO), Vienna (UNIDO),


Paris (UNESCO), Nairobi (UNEP) and the Group of 24 in Washington,
D.C. (International Monetary Fund and World Bank). So, (c) is incorrect.
www.insightsonindia.com
www.insightsias.com
TEST – 5 Solutions

Learning: It was designed to promote its members' collective economic


interests (such as climate change) and create an enhanced joint
negotiating capacity in the United Nations. There were 77 founding
members of the organization, but by 2013 the organization had since
expanded to 134 member countries.

South Africa holds the Chairmanship for 2015.

Q Source: Chapter 4: Page 100: 10th NCERT World History

51. Solution: b)

Learning: The Cartagena Protocol on Biosafety to the Convention on


Biological Diversity is an international agreement on biosafety, as a
supplement to the Convention on Biological Diversity. The Biosafety
Protocol seeks to protect biological diversity from the potential risks
posed by genetically modified organisms resulting from modern
biotechnology.

The Biosafety Protocol makes clear that products from new technologies
must be based on the precautionary principle and allow developing
nations to balance public health against economic benefits.

Visit http://www.moef.nic.in/sites/default/files/treaties/international-
treaties.html for a list of important treaties.

52. Solution: d)

Justification: The need for maintaining world peace was visibly felt
after WW-I. Hence, the LoN came into existence in 1920 as a result of
the Paris Peace Conference that ended the First World War. So, (a) is
correct.

Its primary goals, as stated in its Covenant, included preventing wars


through collective security and disarmament and settling international
disputes through negotiation and arbitration. So, (b) is correct.

Many developing nations like Ecuador, Paraguay joined the league. So,
(d) is incorrect.
www.insightsonindia.com
www.insightsias.com
TEST – 5 Solutions

It was dissolved in 1946 to form United Nations.

Q Source: 10th NCERT World History

53. Solution: d)

Justification & Learning: Infrasound can result from both natural


and human sources:

It sometimes results naturally from severe weather, surf, lee waves,


avalanches, earthquakes, volcanoes, bolides, waterfalls, calving of
icebergs, aurorae, meteors, lightning and upper-atmospheric lightning.

Infrasound can be generated by human processes such as sonic booms


and explosions (both chemical and nuclear), or by machinery such as
diesel engines, wind turbines and specially designed mechanical
transducers (industrial vibration tables). Certain specialized loudspeaker
designs are also able to reproduce extremely low frequencies

Scientists have picked up infrasounds from two tsunamis during 2006 in


the pacific ocean. To study the phenomena systematically the
researchers have recently set up a tsunami infrasound project in Hawaii.
The scientists hope to learn how the giant waves produce infrasound,
which is currently a mystery.

Rhinoceroses communicate using infrasound of frequency as low as 5


Hz. Whales and elephants produce sound in the infrasound range. It is
observed that some animals get disturbed before earthquakes.

Q Source: Chapter 12: Page 170: 9th Science NCERT

54. Solution: d)

Justification: Ecomark is a certification mark issued by the Bureau of


Indian Standards (the national standards organization of India) to
products conforming to a set of standards aimed at the least impact on
the ecosystem.
www.insightsonindia.com
www.insightsias.com
TEST – 5 Solutions

The marking scheme was started in 1991. One of the purposes of the
mark is increasing awareness among the consumers towards reducing
environment impact.

The mark is issued to various product categories and the development of


standards for more products is in progress.

Q Source: http://www.moef.nic.in/public-information/national-
action-programme-combat-desertification-0

55. Solution: d)

Learning: UK has been in news for a variety of reasons (PM’s visit;


demand for Ireland autonomy, monarchical system issues etc.)

The United Kingdom of Great Britain and Northern Ireland, commonly


known as the United Kingdom (UK) or Britain, is a sovereign state in
Europe.

The UK consists of four countries: England, Scotland, Wales, and


Northern Ireland. The latter three have devolved administrations, each
with varying powers,

The nearby Isle of Man, Bailiwick of Guernsey and Bailiwick of Jersey


are not part of the United Kingdom, being Crown dependencies with the
British Government responsible for defence and international
representation.

Q Source: 10th NCERT World History & Contemporary events

56. Solution: b)

Justification: The Equal Remuneration Act, 1976 provides that equal


wages should be paid to equal work for men and women. It was enacted
in pursuance of the DPSP.

Article 39: "The State shall in particular, direct its policy towards
securing -
www.insightsonindia.com
www.insightsias.com
TEST – 5 Solutions

• that the citizen, men and women equally, have the right to an
adequate means of

• livelihood.

• that the ownership and control of the material resources of the


community are so

• distributed as best to subserve the common good.

• that the operation of the economic system does not result in the
concentration of

• wealth and means of production to the common detriment.

• that there is equal pay for equal work for both men and women.

Q Source: Improvisation: Chapter 4: 10th NCERT Polity

57. Solution: a)

Justification: Option (d) is known as ‘social or national polarization’


and is a general term. Hence not appropriate here.

Option (c) is known as feudal fragmentation, hence incorrect.

Option (b) may be a cause of balkanization, but is not known as


balkanization. Hence, incorrect.

Leaning: The term refers to the division of the Balkan peninsula,


formerly ruled almost entirely by the Ottoman Empire, into a number of
smaller states between 1817 and 1912.

In India, the term is often employed to discuss the implications of


extreme regionoalism in the NE and some other parts of the country.
Refer to the article below to understand it.

http://www.asianews.it/news-en/Balkanization-of-India:-a-roadmap-
of-Beijing-16113.html

Q Source: 10th NCERT World History & Contemporary events

58. Solution: d)
www.insightsonindia.com
www.insightsias.com
TEST – 5 Solutions

Justification & Learning: The third law of motion states that when
one object exerts a force on another object, the second object
instantaneously exerts a force back on the first. These two forces are
always equal in magnitude but opposite in direction.

It is important to note that even though the action and reaction forces
are always equal in magnitude, these forces may not produce
accelerations of equal magnitudes.

This is because each force acts on a different object that may have a
different mass.

When a gun is fired, it exerts a forward force on the bullet. The bullet
exerts an equal and opposite reaction force on the gun. This results in
the recoil of the gun. Since the gun has a much greater mass than the
bullet, the acceleration of the gun is much less than the acceleration of
the bullet.

Hence, (d) is correct.

Q Source: Chapter 9: Page 122: 9th Science NCERT

59. Solution: a)

Justification: Below is the map of the Silk Road. We are emphasizing


on Silk road as it is important for Prelims, due to China’s One road One
belt initiative.

The road connected civilizations across Europe, Asia (including Persia,


Arabia) and Africa (only the horn part).

The disappearance of the Silk Road following the end of the Mongols'
reign was one of the main factors that stimulated the Europeans to reach
the prosperous Chinese empire through another route, especially by sea.

The direct ocean route from Europe to the East was finally opened by the
expeditions of Bartolomeu Dias (1488), and Vasco da Gama (1497-1499),
by the Atlantic and the Indian oceans.
www.insightsonindia.com
www.insightsias.com
TEST – 5 Solutions

Q Source: 10th NCERT World History & Contemporary events

60. Solution: d)

Justification: Statement 1: During the war time, most of the wheat


exports went to war zones. Moreover, production in Eastern Europe had
declined. After the war, when production in Eastern Europe resumed, it
led to over-supply and falling prices. Thus income of farmers declined
and this led to an overall decline in the demand, investment and
production.

Statement 2: In the 1920s, most countries financed their investments by


borrowing from the US. US overseas lenders panicked at the economic
slowdown, and withdrew most loans from the market. This withdrawal
of credit affected Europe as its banks collapsed and major currencies
depreciated greatly. Moreover, US hiked import duties to save its
economy. All this dealt a severe blow to the global economy.

Statement 3: Fragile post-war economy: war goods during the FWW had
created most of the production boom. With the war ending, this
production declined, leading to job losses, decline in income and further
www.insightsonindia.com
www.insightsias.com
TEST – 5 Solutions

decline in demand of goods. It created a vicious cycle of low demand, low


investment and unemployment.

Q Source: Chapter 4: Page 95: 10th NCERT World History

61. Solution: a)

Justification & Learning: Ports in India are classified as Major and


Minor Ports according to the jurisdiction of the Central and State
government as defined under the Indian Ports Act, 1908. So, (a) is
correct.

• Major Ports are under the Union list of the Indian Constitution and
are administered under the Indian Ports Act 1908 and the Major
Port Trust Act, 1963. Each major port is governed by a Board of
Trustees appointed by the Government of India.

• The tariffs for major ports are fixed by the Tariff Authority for
Major Ports (TAMP).

• Minor operates are managed at the State level by the department


in charge of ports or the State Maritime Board, if created, as is the
case in Gujarat, Maharashtra and Tamil Nadu.

• The functions of the State maritime boards are similar to those of


port trusts, and also include the authority to set tariffs. They also
focus on attracting private investment by awarding concession
contracts, providing incentives, exclusivity rights and assuring land
acquisition.

Q Source: Yojana November 2015 – Page 19

62. Solution: d)

Justification & Learning: Due to Partition an artificial barrier


between jute supply-demand in India as 81% of existing jute cultivation
was in what became East Pakistan, while all of the mills were in West
Bengal.
www.insightsonindia.com
www.insightsias.com
TEST – 5 Solutions

Moreover, East Pakistan imposed export duties on raw jute, and


selectively reduced the quality of jute shipped to India. Most of the
Jute Industrialist started to evacuate India, leaving behind the industrial
setup of the Jute Industry. The twin results were a sharp decline in the
supply of raw jute to Indian mills, and a subsequent increase in the price
of raw jute until 1950 (remaining near WWII price levels).

Hence, there was a reduction of the quality of the finished product in


India. India’s textile export decreased as the result of these
developments.

Q Source: Chapter 3: 10th NCERT World History

63. Solution: d)

Justification: Even a retail investor (a common man) can invest in


security markets abroad. Also, several stock brokerage firms (operating
within only one nation) can invest all over the World. They are not called
MNCs. So, (a) is wrong.

Similar to the reasoning mentioned above, (b) is wrong.

It is possible for even a domestic firm to raise capital from abroad. It


becomes an MNC only when it operates in more than one nation. So, (c)
is wrong.

Learning: The first MNCs were established in the 1920s. Many more
came up in the 1950s and 1960s as US businesses expanded worldwide
and Western Europe and Japan also recovered to become powerful
industrial economies.

The worldwide spread of MNCs was a notable feature of the 1950s and
1960s. This was partly because high import tariffs imposed by different
governments forced MNCs to locate their manufacturing operations and
become ‘domestic producers’ in as many countries as possible.

Q Source: Chapter 4: Page 100: 10th NCERT World History

64. Solution: c)
www.insightsonindia.com
www.insightsias.com
TEST – 5 Solutions

Justification: Statement 1: The fundamental provisions of the


constitution cannot be unilaterally changed by one level of government.
Such changes require the consent of both the levels of government. For
e.g. Central govt. In India cannot usurp all powers of the states in the
state list of the 7th schedule.

Statement 2: Suppose, if any one government resolved the disputes


between federal constituents, there will be a violation of natural justice.
So, an independent judiciary is needed.

Q Source: Chapter 2: Page 15: 10th NCERT Polity

65. Solution: a)

Learning: In the 18th century, the East India Company had established
itself in India. Indian cotton and silk fabrics were in great demand
worldwide and hence were of special interest to them.

It proceeded to develop a system of management and control that would


eliminate competition, control costs, and ensure regular supplies of
cotton and silk goods. Given the small number of Englishmen, and their
unfamiliarity with the local language and society, the Company turned to
local intermediaries, and gave them legal authority to enforce contracts.

The Company tried to eliminate the existing traders and brokers


connected with the cloth trade, and establish a more direct control over
the weaver. For this purpose they appointed paid servants called the
gomasthas were employed who would obtain goods and from local
weavers and fix their prices.

The prices fixed were 15 per cent lower than market price and in extreme
cases, even 40 per cent lower than the market price.

They would also supervise weavers, collect supplies, and examine the
quality of cloth. They also prevented Company weavers from dealing
with other buyers.

Q Source: Chapter 5: Page 126: 10th NCERT World History


www.insightsonindia.com
www.insightsias.com
TEST – 5 Solutions

66. Solution: a)

Justification: Taking care of innocent children is one of the prime


concerns of PARA. So, (d) is wrong.

CMM rally was about miner’s rights and welfare. So, (b) is also wrong.

There is no movement named unmoolan andolan in WB. So, (c) is


wrong.

Learning: In independent India, sarvodaya activists campaigned for


prohibition. A mass campaign against liquor sale emerged in
Uttarakhand, then part of UP, in the early 1960s. Vimla and Sunderlal
Bahuguna got the government to cancel a contract to sell liquor in their
village.

By 1966, protests spread across the region. Hundreds of picketing


women were jailed. In 1972, the government agreed to impose
prohibition in the Uttarakhand region. Scholars have argued that the
anti-liquor movement politically empowered the region’s women,
leading to their participation in the Chipko movement and, then, the
struggle for a separate Uttarakhand state.

Another notable mass movement against liquor was in Andhra Pradesh


in the early 1990s.

Q Source: Improvisation: Chapter 5: Page 132: 10th NCERT World


History

http://indianexpress.com/article/explained/in-fact-prohibitions-
intoxicating-appeal-for-politicians/

67. Solution: c)

Justification: Option (a) is the like the practice of “initiative” present in


some democratic nations like Canada.

Option (b) is akin to PRIs and Urban Local Bodies (ULBs).

Learning: A referendum (in some countries synonymous with


plebiscite — or a vote on a ballot question) is a direct vote in which an
www.insightsonindia.com
www.insightsias.com
TEST – 5 Solutions

entire electorate is asked to vote on a particular proposal. This may


result in the adoption of a new law.

For e.g. recently in Greece, the government asked the voter to choose an
austerity package for Greece or choose to exit from European Union.

Q Source: Chapter 1: Page 4: 10th NCERT World History

68. Solution: c)

Learning: Mahalanobis strategy of development emphasising basic


heavy industries which was adopted first of all in the Second Plan also
continued to hold the stage in Indian planning right up to the Fifth Plan
which was terminated by the Janata Government in March 1978, a year
before its full term of five years.

In critique of the Mahalanobis heavy industry development strategy,


Professors Vakil and Brahmanand of Bombay University put forward a
wage-goods model of development and suggested a development
strategy which accorded a top priority to agriculture and other wage-
goods industries in sharp contrast to the Mahalanobis heavy industry
biased strategy of development.

So, the essential pillars of Nehru - Mahalanobis strategy were - (a) high
savings rate, (b) heavy industry bias, (c) protectionist policies and public
sector, (d) import substitution, and (e) socialistic pattern of society.

Q Source: http://indianexpress.com/article/opinion/columns/craving-
capital/

69. Solution: b)

Justification: Akbar ruled with a social and religious toleration that


was relative, not absolute, and was based on his concept of sulh-i-kul (for
the general good of all people) which built on his liberal views of religion.
This was however not connected with the book in question. So, (a) is
wrong.
www.insightsonindia.com
www.insightsias.com
TEST – 5 Solutions

Ibadat khana was a place where Akbar used to have religious


discussions. (c) is also wrong.

The book Akhlaq-i Nasiri is divided to three parts: ethics, domestic


economy and politics.

It is considered a medieval treatise on Ismaili Thought.

Q Source: Chapter 7: Page 154: 10th NCERT World History

70. Solution: d)

Justification: Statement 2 and 3 are generally features of socialistic


pattern of economy. Hence, wrong.

Capitalistic economies may have monopolistic as well competitive


markets. However, the core idea is that it should be a free market which
operates without any government interference.

Learning: Capitalism is an economic system based on private


ownership of the means of production and the creation of goods and
services for profit.

Central characteristics of capitalism include private property, capital


accumulation, wage labour and competitive markets.

In a capitalist market economy, investments are determined by private


decision and the parties to a transaction typically determine the prices at
which they exchange assets, goods, and service

Q Source: Chapter 4: 10th NCERT World History

71. Solution: d)

Justification: It was in practice in all these states.

The Jharkhand region (now a state) of India was a hotbed for feudalism.
Feudal lords ruled the region for decades; semi-feudal conditions still
exist. So, 4 is correct.
www.insightsonindia.com
www.insightsias.com
TEST – 5 Solutions

There were a number of feudal states in Kerala in the Middle Ages


between the end of Chera dynasty and the British rule. So, 1 is correct.

Several zamindaris were established in the Madras Presidency (present-


day Tamil Nadu and adjoining areas) from 1799 onwards. The zamindari
settlement was based on an a similar settlement established in Bengal.
So, 3 is correct.

In Maharashtra, Vidharbha region has feudal lords. So, 2 is also correct.

Learning: Use of the term feudalism to describe India applies a concept


of medieval European origin, according to which the landed nobility held
lands from the Crown in exchange for military service, and vassals were
in turn tenants of the nobles, while the peasants (villeins or serfs) were
obliged to live on their lord's land and give him homage, labor, and a
share of the produce, notionally in exchange for military protection.

Q Source: Improvisation: Chapter 1: 10th NCERT World History

72. Solution: d)

Learning: The Vernacular Press Act was passed in 1878 under the
Governor Generalship and Viceroyalty of Lord Lytton, for better control
of Indian language newspapers.

The purpose of the Act was to control the printing and circulation of
seditious material, specifically that which could produce disaffection
against the British Government in India in the minds of the masses. It
was passed on the model of irish press laws.

It provided the government extensive rights to censor report and


editorial in vernacular press.

At the time the Vernacular Press Act was passed, there were thirty five
vernacular papers in Bengal, including the Amrita Bazar Patrika

Q Source: Chapter 7: Page 176: 10th NCERT World History

73. Solution: a)
www.insightsonindia.com
www.insightsias.com
TEST – 5 Solutions

Justification: Bankim Chandra Chattopadhyay wrote the song “Vande


matram” inspired by which Bipin Chandra Pal decided to start a patriotic
journal in August 1906, name vande matram. So, 1 is incorrect and 3 is
correct.

Lala Lajpath Rai started an Urdu daily- “Vande Mataram” from Lahore
which boasted huge circulation.

Its first edition itself was confiscated by police and he had to appeal to
the high court against this. Four editors of this newspaper were sent to
jail under the charge of sedition.

Q Source: Chapter 8: Page 190: 10th NCERT World History

74. Solution: c)

Justification & Learning: It includes exchange of banking


information between tax authorities, but it does not involved amending
laws as such, or giving greater legal powers to banks. So, 2 is incorrect.

It aims at avoidance of double taxation and the prevention of fiscal


evasion with respect to taxes on income. The protocol incorporates
provisions for an effective exchange of information and assistance in
collection of taxes between tax authorities of the two countries.

Q Source:
http://www.thehindubusinessline.com/economy/policy/cabinet-nod-
for-dta-protocol-with-japan/article7941954.ece

75. Solution: a)

Justification: The cracks or holes inside the metal blocks, which are
invisible from outside reduces the strength of the structure. Ultrasonic
waves are allowed to pass through the metal block and detectors are used
to detect the transmitted waves. If there is even a small defect, the
ultrasound gets reflected back indicating the presence of the flaw or
defect. So, 1 is correct.
www.insightsonindia.com
www.insightsias.com
TEST – 5 Solutions

Sonic and ultrasonic weapons (USW) are weapons of various types that
use sound to injure, incapacitate, or kill an opponent. Some sonic
weapons are currently in limited use or in research and development by
military and police forces. So, 2 is correct.

High power ultrasound can break up stony deposits or tissue, accelerate


the effect of drugs in a targeted area, but not deliver drugs. Delivering
drugs can be possible through nanotechnology. So, 3 is incorrect.

Q Source: Chapter 12: Page 170: 9th Science NCERT

76. Solution: b)

Justification: It is not an attempt to counter China’s string of pearls.


So, 1 is wrong.

It does not extend to international waters. So, 2 is wrong.

Learning: The Sagarmala initiative will address these challenges by


focusing on three pillars of development, namely (i) Supporting and
enabling Port-led Development through appropriate policy and
institutional interventions and providing for an institutional framework
for ensuring inter-agency and ministries/departments/states’
collaboration for integrated development, (ii) Port Infrastructure
Enhancement, including modernization and setting up of new ports, and
(iii) Efficient Evacuation to and from hinterland.

It is essentially an infrastructure-cum-policy initiative. It will allow the


central government to have a say in the development of non-major ports
without adopting a confrontationist approach with the state
governments. Necessary institutional framework will be built for the
same.

Q Source: Yojana November 2015 – Page 20

77. Solution: d)
www.insightsonindia.com
www.insightsias.com
TEST – 5 Solutions

Justification: 1% of total highways outlays will be reserved for planting


trees along the highways. It will be counted in the official forest cover.
So, (a) is not correct.

Trees along the highways reduce glare coming from vehicle lights and
sun light, which have been a cause for road accidents. So, (b) is not
correct.

(c) is obvious as trees will act as carbon sink.

It may actually increase the land acquisition as well as it cost. Land


acquisition related issues are not the focal point of policy. So, (d) is the
answer.

Learning: About Rs. 1000 crore per year will be available for plantation
purpose. The policy will generate employment opportunities for about
five lakh people from rural areas.

There will be strong monitoring mechanism in place by using ISRO’s


Bhuvan and GAGAN satellite systems. Every planted tree will be counted
and auditing will be done. The agencies performing well will be awarded.

Q Source: Yojana November 2015 – Page 26

78. Solution: b)

Justification: All objects experience a force of buoyancy upwards when


they are immersed in a fluid. This buoyant force reduces the total
pressure/force the object exerts on a spring balance.

The magnitude of this buoyant force depends on the density of the fluid.
So, the denser the liquid, higher is the buoyant force, and lesser is the
weight experienced.

Sea water has a density of 1022 kg/cubic metres, fresh water has 1000,
ethanol has 786, and lubricating oil has around 900. So, it weighs least
in sea water.

Hence, (b) is correct.

Q Source: Chapter 10: Page 140: 9th Science NCERT


www.insightsonindia.com
www.insightsias.com
TEST – 5 Solutions

79. Solution: d)

Justification: IMF does not peg a currency’s level. It was done earlier
in the Gold system where currencies were fixed in value by some
authority, for e.g. USA government fixed it by an agreement for India
and other nations. So, 1 is wrong.

If a nation follows floating system, all its external transactions are based
on the floating system; similar for the fixed system. So, 2 is wrong.

Learning: Exchange rates – They link national currencies for purposes


of international trade. There are broadly two kinds of exchange rates:
fixed exchange rate and floating exchange rate

Fixed exchange rates – When exchange rates are fixed and governments
intervene to prevent movements in them

Flexible or floating exchange rates – These rates fluctuate depending on


demand and supply of currencies in foreign exchange markets, in
principle without interference by governments.

Q Source: Chapter 4: Page 101: 10th NCERT World History

80. Solution: a)

Justification: IMO is governed by an assembly of members and is


financially administered by a council of members elected from the
assembly.

The IMO's primary purpose is to develop and maintain a comprehensive


regulatory framework for shipping and its remit today includes safety,
environmental concerns, legal matters, technical co-operation, maritime
security and the efficiency of shipping. So, 2 is correct.

These landlocked countries include Afghanistan, Andorra, Armenia,


Belarus, Bhutan, Botswana etc. So, 3 is correct.

Learning: India has been one of the earliest members of the IMO,
having ratified its Convention and joined it as a member-state in the year
www.insightsonindia.com
www.insightsias.com
TEST – 5 Solutions

1959. India has had the privilege of being elected to and serving the
Council of the IMO, ever since it started functioning, and till date, except
for two years for the period 1983-1984.

IMO Council plays a crucial role to play in deciding various important


matters within the mandate of the IMO, in relation to the global shipping
industry, including its work programme strategy and budget.

Q Source: http://pib.nic.in/newsite/PrintRelease.aspx?relid=131979

81. Solution: c)

Justification & Learning: The Diamond Quadrilateral is a project of


the Indian railways to establish high speed rail network in India. This
quadrilateral will connect the four metro cities in India, i.e. Delhi,
Mumbai, Chennai and Kolkata.

This project is similar to Golden Quadrilateral which is a roadway project


which connects the four metros by Express Ways. The Golden
Quadrilateral falls under National Highways Development Project which
has helped to build better road transport in India.

India has planned to start Semi-high speed trains on nine corridors and
has ambition to run bullet trains in future. High-speed train on Mumbai-
Ahmedabad section will be the first bullet train corridor to be
implemented in the country.

Q Source: Yojana November 2015 – Page 67

82. Solution: d)

Justification: You can make the building taller, shorter, wider, flatter,
narrower, circular etc. All of this will have an effect on how sound is
transmitted and felt inside the place. So, 1 is correct.

After production, sound is transmitted and reflected throughout the


building. Changing the building material will change the total absorption
www.insightsonindia.com
www.insightsias.com
TEST – 5 Solutions

/reflection of sound. For e.g. putting more cushion and less metal in the
seats will reduce reflection of sound. So, 2 is also correct.

Audience (people) act as absorbers of sound. If you walk into an empty


hall, an echo can be heard, but rarely in a filled hall. So, 3 is also correct.

Learning: A sound created in a big hall will persist by repeated


reflection from the walls until it is reduced to a value where it is no
longer audible. The repeated reflection that results in this persistence of
sound is called reverberation. In an auditorium or big hall excessive
reverberation is highly undesirable.

To reduce reverberation, the roof and walls of the auditorium are


generally covered with sound-absorbent materials like compressed
fibreboard, rough plaster or draperies. The seat materials are also
selected on the basis of their sound absorbing properties.

Q Source: Chapter 12: Page 168: 9th Science NCERT

83. Solution: c)

Learning: LRSAM is also called Barak 8 missile in Israel which in


Hebrew language means Lightning. Barak 8 is an advanced, long-range
missile defence and air defence system.

Its main features are long range, active radar seeker missile, vertical
launch and multiple simultaneous engagements.

For the LRSAM, DRDO has developed dual pulse propulsion system
along with other safety arm mechanisms for Solid Propulsion system.

The LRSAM programme consists of Missiles, Weapon Control System,


MFSTAR (Radar), Vertical Launcher unit and two- way data link.

It is capable to counter a wide variety of air-borne threats such as anti-


ship missiles, aircraft, UAVs and drones as well as supersonic cruise
missiles.

Q Source: http://www.thehindu.com/todays-paper/tp-national/long-
range-missile-flight-tested-successfully/article7921007.ece
www.insightsonindia.com
www.insightsias.com
TEST – 5 Solutions

84. Solution: b)

Justification: Option (d) is related with Echo.

Option (a) is related with communication in species like bats.

The acoustic frequencies used in sonar systems vary from very low
(infrasonic) to extremely high (ultrasonic). But, SONAR waves do not
cause destruction of sea bed. So, (c) is wrong.

Learning: When the speed of any object exceeds the speed of sound it is
said to be travelling at supersonic speed. Bullets, jet aircrafts etc. often
travel at supersonic speeds. When a sound, producing source moves with
a speed higher than that of sound, it produces shock waves in air.

These shock waves carry a large amount of energy. The air pressure
variation associated with this type of shock waves produces a very sharp
and loud sound called the “sonic boom”. The shock waves produced by a
supersonic aircraft have enough energy to shatter glass and even damage
buildings.

Q Source: Chapter 12: Page 167: 9th Science NCERT

85. Solution: c)

Justification: When a body is immersed fully or partially in a fluid, it


experiences an upward force that is equal to the weight of the fluid
displaced by it.

The weight of the fluid displaced depends on the volume of the object
and its density.

Basically, density of water * volume of water displaced should be =


density of the object * its volume immersed in water.

If you increase the volume it can displace more water, which will exert
greater upwards force on its preventing it from sinking.
www.insightsonindia.com
www.insightsias.com
TEST – 5 Solutions

This concept is used in the design of ships and submarines. So, (c) is
correct, and others are wrong, because neither increased surface area nor
length helps it.

Q Source: Chapter 10: Page 140: 9th Science NCERT

86. Solution: c)

Justification: Here is the oxygen cycle.

If you notice, the only major process returning oxygen is photosynthesis.


This is why afforestation is of even greater importance to tackle climate
change and pollution.

Formation of ozone consumes oxygen. Even the process of nitrogen-


fixing by bacteria does not take place in the presence of oxygen. So, 2 and
3 are incorrect.

Q Source: Chapter 14: Page 200: 9th Science NCERT

87. Solution: c)

Learning: Apart from the ones mentioned in the option, key provisions
of the Bill are:
www.insightsonindia.com
www.insightsias.com
TEST – 5 Solutions

• It seeks to make the life of common man hassle free by providing


services in a time bound manner.
• It also makes it mandatory for each government department, local
body and public authority will issue a ‘Citizen’s Charter’.
• They would set out in detail the time stipulated for provision of
citizen-related services and the officer responsible for providing it.
Otherwise, it Imposes penalty on officials for delay in providing
services.

Q Source: http://www.thehindu.com/news/cities/Delhi/assembly-
clears-bill-on-timebound-delivery-of-services/article7921122.ece

88. Solution: a)

Justification: Option (b) is mixed FARMING. Note the difference.

Option (c) is crop rotation, where crops are not grown simultaneously.

Option (d) is also a kind of mixed farming practice, where food is also
grown side by side.

Learning: Different ways of growing crops can be used to give


maximum benefit.

Mixed cropping is growing two or more crops simultaneously on the


same piece of land, for example, wheat + gram, or wheat + mustard, or
groundnut + sunflower. This reduces risk and gives some insurance
against failure of one of the crops.

Depending upon the duration, crop rotation is done for different crop
combinations. The availability of moisture and irrigation facilities decide
the choice of the crop to be cultivated after one harvest. If crop rotation
is done properly then two or three crops can be grown in a year with
good harvests.

Other such practice is inter-cropping which has been covered in an


earlier test.

Q Source: Chapter 15: Page 208: 9th Science NCERT


www.insightsonindia.com
www.insightsias.com
TEST – 5 Solutions

89. Solution: a)

Justification: It is funded by the Union Ministry of Culture, and has


centres in Bhubaneswar; Chennai; Garhi, Delhi; Kolkata; Lucknow and
Shimla.

In April, 2015, Government of India took over management control of


Lalit Kala Akademi citing several complaints regarding alleged
administrative and financial irregularities in its functioning. So, 3 is
incorrect.

This and other Akademis were recently in news due to the returning of
Sahitya Akademi awards.

You can read the article to understand more about it.

Q Source: http://indianexpress.com/article/explained/writers-protest-
what-returning-sahitya-akademi-honour-means/

90. Solution: a)

Justification: For e.g. India has a major interest in Seychelles. A fair


sized (8%) population of Indian descent, mainly from Tamil Nadu and
Gujarat lives there. So, 1 is correct.

Seychelles could be considered as an `unsinkable aircraft carrier` in the


Indian Ocean given its proximity to the oil sea lanes and oil producing
nations, quite akin to the Ascension island in the Atlantic that the British
used extensively to launch attacks against the Argentinean Navy in the
Falklands war. In fact, China is using it for its string of pearls. So, 2 is
correct.

Majority of oil tankers pass through these regions, not majority of trade.
So, 3 will be incorrect.

Q Source: http://www.narendramodi.in/prime-minister-narendra-
modis-visit-to-seychelles-7316
www.insightsonindia.com
www.insightsias.com
TEST – 5 Solutions

91. Solution: d)

Justification: The mineral nutrients are divided into two groups:


macronutrients and micronutrients.

Macronutrients can be broken into two more groups: primary and


secondary nutrients.

The primary nutrients are nitrogen (N), phosphorus (P), and potassium
(K). These major nutrients usually are lacking from the soil first because
plants use large amounts for their growth and survival.

The secondary nutrients are calcium (Ca), magnesium (Mg), and sulfur
(S). There are usually enough of these nutrients in the soil so fertilization
is not always needed. Also, large amounts of Calcium and Magnesium
are added when lime is applied to acidic soils.

So, (d) is the answer as all others are micro-nutrients.

Learning: The 13 mineral nutrients, which come from the soil, are
dissolved in water and absorbed through a plant's roots. There are not
always enough of these nutrients in the soil for a plant to grow healthy.
This is why many farmers and gardeners use fertilizers to add the
nutrients to the soil.

Q Source: Chapter 15: Page 206: 9th Science NCERT

92. Solution: d)

Justification: Nuclear fusion is a nuclear reaction in which two or


more atomic nuclei come very close and then collide at a very high speed
and join to form a new nucleus.

During this process, matter is not conserved because some of the matter
of the fusing nuclei is converted to photons (energy). So, neutrinos do
not have any role to play as they hardly interact with matter. So, 1 is
incorrect.
www.insightsonindia.com
www.insightsias.com
TEST – 5 Solutions

All reactors in India use nuclear fission technology. Fusion technology


has not been developed for commercial uses at large scales. So, 2 is
incorrect.

Q Source: Xth NCERT

93. Solution: d)

Learning: The molecules of air and other fine particles in the


atmosphere have size smaller than the wavelength of visible light. These
are more effective in scattering light of shorter wavelengths at the blue
end than light of longer wavelengths at the red end. The red light has a
wavelength about 1.8 times greater than blue light.

Thus, when sunlight passes through the atmosphere, the fine particles in
air scatter the blue colour (shorter wavelengths) more strongly than red.
The scattered blue light enters our eyes. If the earth had no atmosphere,
there would not have been any scattering. Then, the sky would have
looked dark. The sky appears dark to passengers flying at very high
altitudes, as scattering is not prominent at such heights. So, the answer
is (d).

Justification: A light pillar is an atmospheric optical phenomenon in


the form of a vertical column of light which appears to extend above
and/or below a light source.

Diffraction refers to various phenomena which occur when a wave


encounters an obstacle or a slit. In classical physics, the diffraction
phenomenon is described as the interference of waves.

Q Source: Xth NCERT

94. Solution: d)

Justification: Eutrophication is the process by which a body of water


acquires a high concentration of nutrients, especially phosphates and
nitrates. These typically promote excessive growth of algae. As the algae
die and decompose, high levels of organic matter and the decomposing
organisms deplete the water of available oxygen, causing the death of
www.insightsonindia.com
www.insightsias.com
TEST – 5 Solutions

other organisms, such as fish. Eutrophication is a natural, slow-aging


process for a water body, but human activity (industrial effluents for e.g.
) greatly speeds up the process. So, 1 is correct.

Heated Industrial effluents increase the temperature of the water body


and typically decrease the level of dissolved oxygen of water. This can
harm aquatic animals such as fish, amphibians and other aquatic
organisms. So, 2 is correct.

A keystone species is a plant or animal that plays a unique and crucial


role in the way an ecosystem functions (e.g. sea star). Statement 3
follows.

Q Source: Xth NCERT

95. Solution: c)

Justification & Learning: The Union Cabinet recently gave its


approval for enactment of Central legislation for declaring 101 additional
Inland Waterways as National Waterways (NW) for navigation. If a
waterway is not declared national waterways, it cannot be used for
development for navigational purposes.

This will create a logistic supply chain with intermodal (Rail, Road and
Waterways) connectivity and "would positively contribute to the GDP by
opening up business opportunities in the area of dredging, barge
construction, barge operation, barge repair facilities, terminal
construction, tourist cruise etc.

• On declaration of a waterway as a NW, development and


regulation for shipping and navigation by mechanically
propelled vessels come under the jurisdiction of Central
Government.
• However, rights over the usage of water, ownership of
appurtenant land, minerals, metals, sand etc continue to be with
the State Government.

Q Source: Yojana November 2015 – Last Cover Page


www.insightsonindia.com
www.insightsias.com
TEST – 5 Solutions

96. Solution: b)

Justification: For rays coming from a distance, convex mirrors always


give an erect, though diminished, image. They also have a wider field of
view as they are curved outwards. So, if you wish to obtain a large image
in a small mirror, you should use convex mirrors. So, (b) is correct.

Learning: If you visit the Agra Fort, try to observe the full-length image
of a distant, tall building/tomb in the wall mirror. To view the tomb
distinctly, you should stand suitably at the terrace adjoining the wall.

Convex mirrors are commonly used as rear-view (wing) mirrors in


vehicles. These mirrors are fitted on the sides of the vehicle, enabling the
driver to see traffic behind him/her to facilitate safe driving. Thus,
convex mirrors enable the driver to view much larger area than would be
possible with a plane mirror.

Q Source: Xth NCERT

97. Solution: a)

Justification: Opium wars are an important even in the fight against


imperialism, hence, the question.

Option (b) refers to the Great Turkish war beginning in 1683 and ending
with the signing of the Treaty of Karlowitz in 1699. The war was a defeat
for the Ottoman Empire, which lost large amounts of territory in Central
Europe. The war was also significant in that it marked the first time
Russia was involved in a western European alliance.

Option (c) refers to Mexican Revolution 1910-20.

Option (d) refers to the Crimean War (which was also in news due to the
Ukraine Crisis 2013-14).

Learning: The disputes included the First Opium War (1839–1842)


and the Second Opium War (1856–1860).

They are important because the wars and events between them
weakened the Qing dynasty and reduced China's separation from the rest
of the world.
www.insightsonindia.com
www.insightsias.com
TEST – 5 Solutions

Q Source: 10th NCERT World History & Contemporary events

98. Solution: d)

Justification: Your weight on earth depends on two things: a)


gravitation force of earth; and b) the centrifugal force caused due to
earth’s rotation, which pushes you away from earth’s surface. The
balance of both forces determines your weight.

Your weight will change only if the speed of rotation (and not revolution)
changes. So, 1 is incorrect.

Also, as described above, it really does not matter how fast you are
travelling on earth, because as long as you are on earth’s surface, you will
experience the same gravitational force. So, 2 is also incorrect.

Your weight will decrease if you move to greater height in stratosphere.


So, 3 is wrong.

Q Source: Improvisation: Chapter 10: Page 131: 9th Science NCERT

99. Solution: c)

Justification & Learning: A reserve currency is one which is being


used widely in international transactions, has a stable value and is freely
usable and convertible. China’s Yuan satisfied all these criteria and thus
IMF declared it to be an elite reserve currency.

With this declaration, nations will prefer holding Chinese Yuan as it


gives more liquidity to their forex reserves. This is because the currency
is now worldwide acceptable for settlement of trade payments. Hence, 1
is correct.

SDR is neither a currency nor a claim on the IMF. It is a potential claim


on the freely usable currencies of IMF members, which holders of SDRs
can obtain in exchange for their SDRs. Its value is based on the basket of
the four major currencies, which now becomes five including Yuan. So, 2
is correct.
www.insightsonindia.com
www.insightsias.com
TEST – 5 Solutions

Q Source: http://indianexpress.com/article/explained/simply-put-
dragon-now-in-basket-how-will-sdr-change/

100. Solution: d)

Justification: Gravity is omnipresent, and it can never be shielded


against. So, (c) is incorrect.

Irrespective of the material used, gravity would not change inside the
spacecraft. So, (a) is also incorrect.

Shooting at high speed will not affect the gravity forces. It will only help
escape arth’s gravity quickly. So, (b) is also incorrect.

Learning: In space, it is possible to create "artificial gravity" by


spinning your spacecraft or space station. When the station spins,
centrifugal force acts to pull the inhabitants to the outside. This process
could be used to simulate gravity. It wouldn't be exactly the same,
though, because large Coriolis forces would also be present, and things
would fall in curves instead of straight lines.

This is sort of like the amusement park ride where you get in a big
cylinder with a lot of people and line up against the walls. Then they spin
the cylinder, producing a force that makes you feel pressed up against
the sides. Everyone becomes glued to the walls of the chamber, and then
they drop the floor out. No one falls to the ground because they are being
held to the edges by a force due to rotation.

Q Source: Improvisation: Chapter 10: Page 131: 9th Science NCERT




INSIGHTS ON INDIA MOCK PRELIMINARY EXAM - 2016


INSIGHTS ON INDIA MOCK TEST – 05

GENERAL STUDIES

PAPER-I
Time Allowed: 2 Hours Maximum Marks: 200

INSTRUCTIONS
1. IMMEDITELY AFTER THE COMMENCEMENT OF THE EXAMINATION, YOU SHOULD
CHECK THAT THIS TEST BOOKLET DOES NOT HAVE ANY UNPRINTED OR TORN OR MISSING
PAGES OR ITEMS, ETC. IF SO, GET IT REPLACED BY A COMPLETE TEST BOOKLET.
2. You have to enter your Roll Number on the Test
Booklet in the Box provided alongside. DO NOT
Write anything else on the Test Booklet.
4. This Test Booklet contains 100 items (questions). Each item is printed only in English. Each item
comprises four responses (answers). You will select the response which you want to mark on the
Answer Sheet. In case you feel that there is more than one correct response, mark the response which
you consider the best. In any case, choose ONLY ONE response for each item.
5. You have to mark all your responses ONLY on the separate Answer Sheet provided. See directions in
the Answer Sheet.
6. All items carry equal marks.
7. Before you proceed to mark in the Answer Sheet the response to various items in the Test Booklet, you
have to fill in some particulars in the Answer Sheet as per instructions sent to you with your Admission
Certificate.
8. After you have completed filling in all your responses on the Answer Sheet and the examination has
concluded, you should hand over to the Invigilator only the Answer Sheet. You are permitted to take
away with you the Test Booklet.
9. Sheets for rough work are appended in the Test Booklet at the end.
10. Penalty for wrong answers :
THERE WILL BE PENALTY FOR WRONG ANSWERS MARKED BY A CANDIDATE IN THE
OBJECTIVE TYPE QUESTION PAPERS.
(i) There are four alternatives for the answer to every question. For each question for which a
wrong answer has been given by the candidate, one-third of the marks assigned to that
question will be deducted as penalty.
(ii) If a candidate gives more than one answer, it will be treated as a wrong answer even if one of
the given answers happens to be correct and there will be same penalty as above to that
question.
(iii) If a question is left blank, i.e., no answer is given by the candidate, there will be no penalty for
that question.

http://www.insightsonindia.com

INSIGHTS ON INDIA MOCK TEST SERIES FOR CIVIL SERVICES PRELIMINARY EXAM 2016

http://www.insightsonindia.com INSIGHTS Page 1





1. The Udaipur Solar Observatory (USO) international economic system in


would do a detailed study of the Solar favour of Third World countries.
activity including its magnetic field to
facilitate space weather predictions in
the future. The observatory has been 3. Consider the following about Snow
situated on an island in the middle of a Leopard.
lake. Why? 1. It is listed as endangered on the
a) Middle of the water body, there will IUCN Red List of Threatened
be less turbulence in the air mass Species.
and thereby image quality and 2. It is the National Heritage Animal
seeing will be improved of Afghanistan and Pakistan.
b) In the Island, the cosmic radiation 3. 2015 has been designated as the
will be comparatively lesser than International Year of the Snow
that on land Leopard in selected countries.
c) Measurement of magnetic field
near water body is enhanced as Select the correct answer using the codes
water molecules have zero below.
magnetism, whereas land particles
a) 1 and 2 only
may have some magnetism
b) 2 and 3 only
d) There will be less interference from
c) 1 only
electromagnetic radiation from
d) All of the above
telecommunications towers and
other magnetic objects

4. What differentiates a ‘nation-state’


from a ‘state’?
2. What do you understand by the term
a) A nation-state does not have a
“New International economic order
centralized authority as a state.
(NIEO)” of the 1970s?
b) A state is a mix of several
a) It was a call by developed nations to
ethnicities and communities unlike
give equal space to newly
a nation-state.
decolonized nations in global
c) A nation-state does not have a
economic assistance.
constitution unlike a state.
b) It was a movement by International
d) State is only a legal/political entity,
Economic Association (IEA) to stop
whereas nation-state is a
the exploitation of third world
legal/political entity as well as a
countries by MNCs.
cultural/social entity.
c) It was a set of proposals by African
nations to stop colonial conquest
and economic exploitation.
d) It was a call by some developing
nations for revision of the

http://www.insightsonindia.com INSIGHTS Page 2






5. Consider the following about State Select the correct answer using the codes
Biodiversity Boards (SBBs). below.
1. It regulates commercial utilization
of biological resources of India. a) 1 and 2 only
2. It is responsible for the b) 2 and 3 only
conservation of folk varieties and c) 1 only
cultivars in forests. d) None of the above

Which of the above is/are correct?

a) 1 only 8. Why is the gravitational force of moon


b) 2 only lesser than earth?
c) Both 1 and 2 1. Moon has no atmosphere.
d) None 2. Moon spins at a slower rate than
earth.

6. The Bretton Woods institutions were Which of the above is/are correct?
established with which of the following a) 1 only
objectives? b) 2 only
1. Finance post-world war II c) Both 1 and 2
reconstruction d) None
2. To deal with external surpluses and
deficits of member nations 9. If RBI changes the Repo rate, it also
3. To ensure full employment in changes which of the following
developed countries monetary policy tools?
Select the correct answer using the codes 1. Reverse Repo rate
below. 2. Statutory Liquidity Ratio (SLR)
3. Cash Reserve ratio (CRR)
a) 1 and 2 only
b) 2 and 3 only Select the correct answer using the codes
c) 1 only below
d) All of the above a) 2 and 3 only
b) 2 only
c) 1 and 3 only
7. Which of the following are the d) 1 only
‘disadvantages’ of inland waterways as
compared to roadways? 10. If India declares a national war against
1. It is less fuel efficient. the Islamic state (ISIS), it must be
2. Cost of developing waterways is approved by
generally more than roadways. a) Ministry of External Affairs
3. Overall transportation cost is b) President of India
generally more. c) Indian Armed Forces
d) National Security Council

http://www.insightsonindia.com INSIGHTS Page 3






11. Consider the following statements. Which of the above is/are true?
1. In Railways, investment is done by
both Central and State a) 1 only
governments. b) 2 only
2. In Civil Aviation, investment is c) Both 1 and 2
done both by Central government d) None
and private sector.
14. Consider the following about Bihu
Which of the above is/are correct? dance.
1. It is performed by both young men
a) 1 only and women.
b) 2 only 2. Hand and leg movement is slow as
c) Both 1 and 2 dance is performed on grass plates.
d) None 3. The dance is performed to wish for
good rains and crops.
12. In India, labour laws generally cover
which of the following areas of Select the correct answer using the codes
concern? below.
1. Dispute settlement between the
labourers and management a) 1 and 2 only
2. Provision for hiring and firing of b) 2 and 3 only
labour c) 1 only
3. Provisions regarding payment of d) All of the above
wages
4. Minimum labour qualifications for 15. Funds under the PM Khanij Kshetra
heavy industries Kalyan Yojana (PMKKKY) can be
utilized for
Select the correct answer using the codes 1. Improving physical infrastructure
below of the region
2. Subsidizing economically non-
a) 2 and 4 only viable mining units
b) 1, 2 and 3 only 3. Enhancing quality of environment
c) 1 and 4 only in mining districts
d) All of the above
Select the correct answer using the codes
below.
13. Which of the following are reason(s) a) 1 and 2 only
for non-adoption of bio-toilets at a b) 2 and 3 only
large scale in India? c) 1 and 3 only
1. It cannot work in cold Himalayan d) All of the above
regions.
2. To maintain the health of bacteria,
bio-toilets need to be maintained
regularly.

http://www.insightsonindia.com INSIGHTS Page 4






16. Why are whales and dolphins
mammals and not most other fishes?
1. Like mammals, whales and 19. Consider the following about
dolphins breathe air through a pair ‘Association for Democratic Reforms
of lungs. (ADR)’
2. Like most mammals, they are cold- 1. It is an autonomous body under the
blooded. ECI.
3. Like most mammals, they give 2. It is aided by the government.
birth to live young ones. 3. The ECI relies on its criminal
record findings to approve the
Select the correct answer using the codes nomination of political candidates.
below.
Select the correct answer using the codes
a) 1 and 2 only below.
b) 2 and 3 only
c) 1 and 3 only a) 1 and 2 only
d) 1 only b) 2 only
c) 1 and 3 only
17. Russia has recently imposed economic d) None of the above
sanctions against Turkey on the issue
of 20. The apparent flattening of the Sun’s
a) Turkey has shot down a Russian disc at sunrise and sunset is due to
aircraft which entered its territory 1. Formation of mirage on ground
b) Turkey supporting the Islamic State 2. Halo effect of the Sun
(ISIS) in its territory 3. Cloud iridescence
c) Turkey was suspected to be Select the correct answer using the codes
involved in the development of below.
chemical weapons
d) Turkey was suspected to have a) 2 and 3 only
instigated a coup in the Ukrainian b) 1 and 2 only
region c) 3 only
d) None of the above
18. One of the criteria for recognizing a
political party in a State by the ECI is if 21. Which of the following are convincing
a) It meets criteria of minimum assets arguments for adopting democracy?
in form of land and bank account 1. Democratic nations have had a
liquidity higher economic growth rate
b) It has won at least three local body historically than dictatorial
elections since its formation regimes.
c) It has been engaged in political 2. Only democratic nations can
activity for a continuous period of become members of the United
five years. Nations (UN).
d) At least one of the members of the
Which of the above is/are correct?
party has been an MP for two terms

http://www.insightsonindia.com INSIGHTS Page 5






a) 1 only 2. Act as an indicator of water
b) 2 only pollution.
c) Both 1 and 2 3. Indicate the presence of natural gas
d) None deposits in a certain area

Select the correct answer using the codes


below.
22. Artificial satellites and space probes
like Mars orbiters generally use which a) 1 and 2 only
source of energy? b) 2 only
a) They exploit Dark energy of the c) 2 and 3 only
outer space d) 1 and 3 only
b) They obtain energy from the
microwaves and ultrasonic waves
c) They harness energy from 25. In India, the ‘Constitution’ of political
gravitational waves parties must be approved by
d) They get solar energy from the use a) The Chief Election Commissioner
of solar cells of India
b) Election Commission of India in
case of national parties, and Chief
23. Consider the following about IBSA, an Secretary of the state in case of
international grouping of India, Brazil state parties
and South Africa. c) All State Election Commissions in
1. It originated through the Brasilia which they fight elections
declaration. d) None of the above
2. India has not hosted any IBSA
summit till date. 26. Some hybrid varieties of crops are
3. IBSA members are the largest developed to tackle “biotic and abiotic
democracies of their respective stresses”. What do you understand by
continents. this?
4. It runs the IBSA Facility for Hunger 1. By preventing the plant to
and Poverty Alleviation. biologically choose its own
agronomic characteristics
Select the correct answer using the codes 2. Crop can tackle conditions of
below. greater than normal cold or frost
a) 1 and 4 only 3. Making the crop insect-resistant
b) 2 and 3 only Select the correct answer using the codes
c) 1, 3 and 4 only below.
d) All of the above
a) 1 and 2 only
24. Coliform bacteria can b) 2 and 3 only
1. Spread Tuberculosis among c) 1 and 3 only
humans d) All of the above

http://www.insightsonindia.com INSIGHTS Page 6






27. The SEBI-FMC merger will allow 30. Consider the following about the Atal
Securities and Exchange Board of Mission for Rejuvenation and Urban
India (SEBI) to Transformation (AMRUT) scheme
a) Monitor speculation in stock 1. The scheme can be implemented
market with public private partnership
b) Track and take action against model (PPP) model.
malpractices in commodity futures 2. It will be funded jointly by the
market Centre and States.
c) Fine retailers and firms involved in 3. Projects will be executed by Urban
inflating prices due to hoarding Local Bodies (ULBs).
d) Regulate futuristic mergers of food
related corporations in India Select the correct answer using the codes
below.
28. In areas with light soils, crops get a) 1 and 2 only
adversely affected by drought b) 2 only
conditions. Why? c) 2 and 3 only
a) Light soils have less water retention d) 1, 2 and 3
capacity.
b) Light soils have low iron content. 31. Apart from bacteria, which of the
c) Light soils cannot be irrigated following living species may be
unlike dense soils. involved in the nitrogen cycle in our
d) Light soils change their mineral atmosphere?
nutrients composition on 1. Animals
dehydrating. 2. Green plants
3. Humans

29. United Nations Educational, Scientific Select the correct answer using the codes
and Cultural Organization (UNESCO) below.
Fellini Medal is awarded for a) 1 and 2 only
a) Extraordinary work of courage in b) 2 and 3 only
preservation of natural heritages c) 1 and 3 only
b) Publishing scientific literature of d) All of the above
value to better conserve ancient
heritages 32. Why Ozone does not contribute to
c) Artists who have created cultural warming of atmosphere near to Earth’s
work of astounding value to surface?
humanity a) It is not stable near Earth’s surface.
d) Rewarding notable efforts to boost b) It can contribute to warming only
the art of film where weather phenomenon is
absent.
c) Sun’s rays near earth’s surface do
not contain ultraviolet rays.
d) All of the above
http://www.insightsonindia.com INSIGHTS Page 7




33. What is Advance Pricing Agreements 36. Which of the following factors did NOT
(APAs) signed recently between Indian contribute/propel the Vietnam War?
authorities and several MNCs? a) Anti-communist agenda
a) An agreement to regulate futures b) Fight against colonialism
trading by the MNCs c) North Korean acquisition of
b) An agreement to restrict the Vietnam
involvement of MNCs in security d) Coup in Vietnam government
markets and speculation
c) An agreement for appropriating
transfer pricing methodology. 37. Under the Global Initiative of
d) An agreement to restrict upwards Academic Networks (GIAN) Scheme
revision of price of sensitive 1. Public institutions will be
commodities in India subsidized to engage in cutting
edge research with global
34. You are verbally communicating with universities.
your friend who is separated from you 2. New Indian institutions will be
by a large space which can be filled established abroad in collaboration
with any one of the following at a time. with high level international
1. Distilled Water research organs.
2. Hydrogen
3. Sea water Which of the above is/are correct?

Your friend is likely to hear you earliest and a) 1 only


latest in which mediums respectively? b) 2 only
c) Both 1 and 2
a) 1 and 2 d) None
b) 2 and 3
c) 3 and 2
d) 1 and 3
38. Consider the following about the force
of gravity.
1. It plays an important role in
35. Apart from vehicles, Bharat stage (BS) formation of tides.
norms are emission standards for 2. It is applicable to massless
1. Industrial chimneys particles.
2. Mining corporations 3. It is an important factor in deciding
3. Natural gas exploration companies the shape of a minor planet.
Select the correct answer using the codes Select the correct answer using the cods
below. below.
a) 1 and 2 only a) 1 and 2 only
b) 2 only b) 2 and 3 only
c) 3 only c) 3 only
d) None of the above d) 1 and 3 only

http://www.insightsonindia.com INSIGHTS Page 8






42. Consider the following about the
National Highways Authority of India.
39. Which of the following tiers of 1. It has been established by an act of
government is constitutionally Parliament.
authorized to issue ordinances on 2. It is an autonomous agency of the
Municipalities? Government of India.
a) Central government 3. Only NHAI has the authority to
b) State government approve PPP projects for national
c) Zila Parishad highways.
d) Any of the above
Select the correct answer using the codes
below.
40. In a federal structure, power is usually a) 1 and 2 only
divided across which of the following b) 2 and 3 only
categories? c) 2 only
1. Legislative d) None of the above
2. Financial
3. Administrative
4. National military
5. Diplomatic 43. ASW Corvette Kadmatt of the Indian
Navy is
Select the correct answer using the codes a) A sophisticated RADAR system
below. fitted near lighthouses to detect
pirate activities
a) 1, 2, 4 and 5 only b) A fleet of ships tied together
b) 2 and 4 only technologically to act as first
c) 1, 3 and 5 only defence against a costal attack
d) All of the above c) A submarine that generates in-
house nuclear power and can stay
for long periods near sea beds
41. The ‘Keynesian revolution’ is related to d) None of the above
a) A critic of the free market economic
system
b) Dynamic role of agriculture in 44. Which of the following developments
industrial development could be causal factors behind World
c) The theory of mercantilism which War-I?
advocates increasing wealth by 1. Pan-Slav movement
more exports 2. Race for colonies amongst
d) Development of a centrally planned European empires
economic system 3. Truman Doctrine

Select the correct answer using the codes


below.

http://www.insightsonindia.com INSIGHTS Page 9






a) 1 and 2 only Select the correct answer using the codes
b) 2 and 3 only below.
c) 1 and 3 only
d) All of the above a) 2 and 3 only
b) 2 and 4 only
45. Fungi can be used for c) 1 and 3 only
1. Therapeutic purposes d) 2 only
2. Pest control in agricultural fields
3. Bioremediation 48. What do you understand by the term
4. Improving the photosynthetic ‘Captive power plant’?
efficiency of crops a) Privately managed power plants
that are subsidized by government
Select the correct answer using the codes in national interest
below. b) Power plants established near sea
ports and harbours
a) 1, 2 and 3 only c) A power plant set up by an
b) 3 only individual or an industry generally
c) 1, 2 and 4 only for private use
d) All of the above d) A power plant that is connected to
all grids in the country

46. Which of the following schemes


has/have been subsumed under 49. Which of the following have a bearing
“Swachha Bharat Abhiyaan”? on environmental conservation in
1. JNNURM India?
2. Slum Development Scheme 1. DPSP
3. National Service Scheme 2. Fundamental Rights
Select the correct answer using the codes 3. 73rd amendment
below. 4. 74th amendment

a) 1 and 2 only Select the correct answer using the codes


b) 2 and 3 only below.
c) 2 only a) 1 only
d) None of the above b) 1, 3 ad 4 only
c) 2 and 4 only
47. Apart from carbon dioxide, which of d) 1, 2, 3 and 4
the following gases is
required/produced/absorbed by plants
in the process of photosynthesis?
1. Carbon monoxide
2. Oxygen
3. Nitrogen dioxide
4. Hydrogen

http://www.insightsonindia.com INSIGHTS Page 10






50. Which of the following about G-77, a 53. Which of the following can produce
coalition of developing nations, is infrasound?
correct? 1. Earthquakes
a) India and China are the co- 2. Avalanches
founders of G-77. 3. Tsunamis
b) It founded the Non-Alignment 4. Volcanoes
movement (NAM), 1961. 5. Diesel engines
c) Being an informal grouping, it is 6. Aurorae
officially not recognized by bodies
of United Nations. Select the correct answer using the codes
below.
d) None of the above
a) 4, 5 and 6 only
b) 1, 2 and 3 only
c) 1, 2, 4, 5 and 6 only
51. Which of the following treaties have d) All of the above
NOT been ratified by India?
1. Stockholm Convention on 54. Ecomark Scheme of India is related
Persistent Organic Pollutants with
(POPs) 1. Marking densely forested regions
2. Cartagena Protocol on Biosafety with GPS for better space
3. Rotterdam Convention monitoring
2. Creating eco sensitive areas (ESAs)
Select the correct answer using the codes
around biosphere reserves
below.
Which of the above is/are correct?
a) 1 and 2 only
b) 1 only a) 1 only
c) 3 only b) 2 only
d) All have been ratified. c) Both 1 and 2
d) None

52. Which of the following statements 55. United Kingdom (UK) consists of
about the League of Nations is which of the following regions?
INCORRECT? 1. Greenland
a) It was the first international 2. Wales
organisation whose principal 3. Northern Ireland
mission was to maintain world 4. Scandinavia
peace.
Select the correct answer using the codes
b) It aimed at collective security and below.
disarmament.
c) USA did not join the league. a) 1 and 2 only
d) Its membership was open only to b) 2, 3 and 4 only
c) 1 and 3 only
industrially developed nations.
d) 2 and 3 only

http://www.insightsonindia.com INSIGHTS Page 11






56. The provision for equal wage to man 59. Apart from China and India, trade on
and woman for equal work is given in the Silk Road was a significant factor
1. Fundamental Duties in the development of the civilizations
2. Directive Principles of State policy of
3. The Equal Remuneration Act, 1976 1. Europe
2. Horn of Africa
Select the correct answer using the codes 3. Scandinavia
below. 4. West Africa
a) 1 and 2 only Select the correct answer using the codes
b) 2 and 3 only below.
c) 1 and 3 only
d) All of the above a) 1 and 2 only
b) 1, 3 and 4 only
57. ‘Balkanization’ is a term that is often c) 2 and 4 only
used in political discourse. It means d) None of the above
a) Fragmentation of a state into
smaller units that can be hostile
with one another 60. Which of the following could NOT
b) Excessive centralization of have contributed to Great Depression
authority away from satellite states of 1930s?
of a nation 1. Agricultural over-production
c) A stage in the development of 2. Credit contraction by USA
certain feudal states 3. Fragile post-World War I economy
d) Segregation within a nation that
may emerge from economic Select the correct answer using the codes
displacements below.

a) 1 and 2 only
58. When a gun is fired, it exerts a forward
b) 2 and 3 only
force on the bullet. But, the pace at
c) 1 only
which the gun recoils is much lesser
d) All factors could have contributed.
than that of bullet. What can be the
reason for this?
61. Ports in India are classified as Major
a) The bullet is being shot from the
and Minor Ports according to
barrel of the Gun, which is its
a) Jurisdiction of Central or State
source.
government
b) The Gun extends even behind the
b) Whether they are located in a major
bullet in position, so it recoils
or minor town
lesser.
c) The tariffs charged at the ports
c) Bullet is made from frictionless
d) The destinations both domestic and
material and then shot, whereas the
international that they serve
Gun is held in hand
d) Gun has larger mass than the bullet

http://www.insightsonindia.com INSIGHTS Page 12






62. How was the Jute industry affected
after partition in India? 65. ‘Gomasthas’ in pre-independent India
1. All the jute mills went to Eastern referred to
Pakistan. a) An Indian agent of the British East
2. Majority of jute growing areas in India Company employed to sign
India went to Eastern Pakistan. agreements with local labour
3. No marketing system for Jute b) The local zamindars responsible for
existed in India thereafter and had collection of revenue for the British
to be created afresh. treasury
c) Earliest Indian entrepreneurs that
Select the correct answer using the codes lobbied for ‘Discriminating
below. Protection’ by the British
a) 1 and 2 only d) A union of merchants that financed
b) 2 and 3 only the earliest Indian cotton industries
c) 1 only
d) 2 only 66. The Temperance movement of 19th
century Britain and America was a
middleclass-led social reform
movement against the social evil of
63. Multinational corporations (MNCs)
alcoholism. It draws a parallel in India
are those entities which
with the popular
a) invest in security markets of a
a) Sarvodaya campaign against
nation abroad
alcohol of 1960s
b) invest in sovereign bonds of at least
b) Chhattisgarh Mukti Morcha (CMM)
two or more nations
rally of 1980s
c) are allowed to raise funding from
c) Unmoolan Andolan in West Bengal
abroad
1970s
d) None of the above
d) Peoples Action for Rural
Awakening (PARA) 1980s
64. Which of the following provisions
ensure that the federal structure of a
polity is maintained?
1. Constitution cannot be amended 67. Plebiscite refers to the process where
completely/fundamentally by any a) Citizens of a country can propose
one level of government. legislation for adoption by the
2. There is an independent judiciary legislature
which resolves disputes between b) Local citizen bodies monitor
the federal organs. important government decisions
c) Citizens vote to accept or reject a
Which of the above is/are correct?
proposal
a) 1 only d) Citizens can amend the constitution
b) 2 only of a nation by a special procedure
c) Both 1 and 2
d) None

http://www.insightsonindia.com INSIGHTS Page 13






68. Mahalanobis model of post- 71. Until Independence, ‘Feudalism’ was
independent India stressed upon NOT in practice in India in which of
a) Promoting economic growth the following regions/states?
through the production of wage- 1. Kerala
goods 2. Maharashtra
b) Promoting an export-led growth for 3. TamilNadu
the Indian economy 4. Jharkhand
c) Promoting capital (heavy) intensive
economic growth Select the correct answer using the codes
d) Promoting tertiary and agricultural below
sector over the Industrial sector a) 2 and 3 only
b) 1 only
c) 1, 3 and 4 only
69. Akhlaq-i Nasiri is a d) None of (a), (b) or (c) is correct.
a) Part of the overall code of conduct
of Akbar’s Sulh-i-Kul religion
b) Persian book in philosophical ethics 72. The Vernacular Press Act in British
written by Al-Tusi India was concerned with
c) A sufi practice of extreme a) Promoting local language
asceticism followed by penance newspapers for generating local
d) A group of scholars in Akbar’s employment
Ibadat Khana who wrote religious b) Providing for licensing and
codes registration of newspapers
published by indigenous tribes
c) Banning all vernacular language
70. Which of the following are NOT the publications in India
general features of ‘capitalistic’ d) None of the above
economies?
1. Competitive markets
2. Centrally planned economy 73. In pre-independent India, journal with
3. Public ownership of labour and the name “Vande Mataram” was
capital started/ published by
Select the correct answer using the codes 1. Bankim Chandra Chattopadhyay
below. 2. Lala Lajpat Rai
3. Bipin Chandra Pal
a) 1 and 3 only
b) 2 only Select the correct answer using the codes
c) 1 only below
d) 2 and 3 only a) 2 and 3 only
b) 2 only
c) 1 and 3 only
d) 1 only

http://www.insightsonindia.com INSIGHTS Page 14






2. Deepening Straits of Malacca to
increase frequency of shipping to
74. The recent cabinet approval of signing India
of Double Taxation Avoidance 3. Port-led development of coastal
Convention (DTAC) between India and areas
Japan will help in
1. Improving the investment climate Select the correct answer using the codes
in India below.
2. Amending banking laws to give
banks legal power for cracking a) 1 and 2 only
down on money laundering b) 3 only
3. Getting mutual assistance in c) 1 and 3 only
matters related to collection of d) None of the above
revenue
77. The Green Highways Policy will NOT
Select the correct answer using the codes help in
below a) Increasing the forest cover of India
b) Reducing road accidents
a) 2 and 3 only c) Cutting down pollution
b) 2 only d) Reducing the land acquisition
c) 1 and 3 only required for road construction
d) 1 only

78. If you use a spring balance to weigh an


75. Apart from determination of foetus, object kept in different mediums, it
which of the following can be the will weigh least in
applications of Ultrasound? a) Ethanol
1. To detect cracks and flaws in metal b) Sea water
blocks c) Lubricating Oil
2. In sonic weapons d) Freshwater lake
3. To deliver drugs directly to the cells

Select the correct answer using the codes


below. 79. What is/are the difference(s) between
a fixed exchange rate and floating
a) 1 and 2 only exchange rate?
b) 2 and 3 only 1. Fixed exchange rate is fixed by the
c) 1 and 3 only IMF whereas floating rate is left to
d) None of the above the markets.
2. Trade of a nation is done based on
76. The Sagarmala project of the floating exchange rate whereas
Government of India focuses on foreign investment is done based
1. Construction of defence checkposts on fixed exchange rates.
in Indian Ocean littoral nations
Which of the above is/are correct?

http://www.insightsonindia.com INSIGHTS Page 15






a) 1 only a) 1 and 2 only
b) 2 only b) 2 and 3 only
c) Both 1 and 2 c) 1 and 3 only
d) None
d) All of the above
80.Consider the following about the
83. The Long Range Surface-to-Air Missile
International Maritime Organization
(LRSAM) co-developed by India and
(IMO).
Israel was recently tested. Consider the
1. It is a specialised agency of the
following about the missile.
United Nations.
1. It can counter air attacks from
2. It is responsible for regulating
drones and anti-ship missiles.
shipping in international waters.
2. It is an active radar seeking missile.
3. Most states that are not members
of IMO are landlocked countries. Which of the above is/are correct?
Select the correct answer using the codes a) 1 only
below b) 2 only
c) Both 1 and 2
a) 1, 2 and 3
d) None
b) 2 only
c) 1 and 3 only
d) 1 only 84. Sonic boom is a phenomenon related
with
81. Diamond quadrilateral project of a) Ultrasonic sounds produced by
Indian railways intends to small vertebrates
a) connect all seven North-eastern b) Supersonic sounds produced by
states with Eastern nations moving objects
b) connect four most important c) Destruction of sea bed due to
mining regions in the country by SONAR waves
way of a freight network d) Repeated reflection of sound in a
c) connect four metro cities by high closed space that increases its effect
speed rail networks manifold
d) develop rail-road linkages in four
farthest corners of the country 85. A large gold nail usually sinks in water.
What can be done to prevent it from
sinking?
82. The acoustics of a theatre can be a) Increasing its height or length to
changed by cover greater area of water for the
1. Changing its architecture given weight
2. Changing its building material b) Decreasing its width to experience a
3. Reducing or increasing the sideward uplifting pressure from
audience sitting in the place water
c) Increasing its total volume for the
Select the correct answer using the codes given weight
below. d) Reducing its surface area for the
given weight
http://www.insightsonindia.com INSIGHTS Page 16




89. Consider the following about the Lalit
86. Oxygen is returned to the atmosphere Kala Akademi or National Academy of
in which of the following processes? Art
1. Photosynthesis 1. It is an autonomous organisation.
2. Formation of Ozone 2. It has the mandate of performing
3. Nitrogen-fixation by legume research in the field of arts.
bacteria 3. Government cannot take the
Select the correct answer using the codes management control of the
below. Akademi.

a) 1 and 2 only Select the correct answer using the codes


b) 2 and 3 only below.
c) 1 only
d) 1 and 3 only a) 1 and 2 only
b) 1 and 3 only
87. The Delhi Right of Citizen to Time c) 2 and 3 only
Bound Delivery of Services d) All of the above
Amendment Bill, 2015 as passed by the
Delhi assembly
1. Ensures mandatory and automatic
90. How are the littoral nations in Indian
compensation for citizens if
Ocean of strategic significance to
government services are not
India?
provided to them in time
1. They have a significant proportion
2. Imposes penalty on officials for
of Indian diaspora.
delay in providing services.
2. Littoral nations are being used for
Which of the above is/are correct? building naval and military bases
by other nations.
a) 1 only 3. Majority of India’s shipping trade
b) 2 only passes through these littoral
c) Both 1 and 2 nations.
d) None
Select the correct answer using the codes
below.
88. Mixed cropping is the practice of a) 1 and 2 only
a) Growing two or more crops b) 1 and 3 only
simultaneously on the same piece c) 2 and 3 only
of land d) All of the above
b) Growing crops and rearing
livestock simultaneously
c) Growing different crops on a piece
of land in a pre-planned succession
d) Growing fodder crops alongwith
livestock

http://www.insightsonindia.com INSIGHTS Page 17






91. Which of the following are plant 94. How industrial effluents dumped in
macro-nutrients? water body may be a threat to aquatic
1. Iron life?
2. Copper 1. By increasing the nutrient content
3. Chlorine of the water
4. Nitrogen 2. Leading to depletion of oxygen
levels of the water body
Select the correct answer using the codes 3. Disrupting the aquatic food chain
below. by killing keystone species
a) 1 and 2 only Select the correct answer using the codes
b) 2, 3 and 4 only below.
c) 1 and 4 only
d) 4 only a) 1 and 2 only
b) 2 and 3 only
c) 1 and 3 only
92. Consider the following about ‘Nuclear d) All of the above
fusion’.
1. It is triggered by fundamental
particle neutrinos. 95. The Central government can legally
2. Few nuclear reactors operational in undertake the development of an
India use this technology. inland waterway for navigation only
when
Which of the above is/are correct? a) It falls under the major-basins of
a) 1 only the country
b) 2 only b) It is connected to a major port of
c) Both 1 and 2 national importance
d) None c) It has been declared as a national
waterway
d) It does not border any tribal areas
falling under the fifth and sixth
93. Why does the sky appear dark instead
schedule
of blue to an astronaut?
a) Due to Light pillar effect
b) Due to absence of diffracted
interference of light 96. You can see a full-length image of a tall
c) Due to absence of dispersion and building/tree in a
diffusion a) Concave Mirror
d) None of the above b) Convex Mirror
c) Plane Mirror
d) Hydraulic Mirrors

http://www.insightsonindia.com INSIGHTS Page 18






97. ‘Opium wars’ of the mid-19th century 99. International Monetary Fund (IMF)
are related to (IMF) has included China’s Yuan into
a) Anglo-Chinese disputes over British its elite reserve currency basket. What
trade in China and China's implications can it have on the
sovereignty International reserve asset system?
b) Conflicts between the Ottoman 1. Nations will now prefer holding
Empire and European powers more Yuan in their forex Reserves.
joined into a Holy League 2. Yuan will now be used for
c) A major armed struggle that calculation of the value of a SDR.
radically transformed Mexican
politics and society Which of the above is/are correct?
d) Rights of Christian minorities in a) 1 only
Mediterranean, which was b) 2 only
controlled by the Ottoman Empire c) Both 1 and 2
d) None

98. Your weight on earth can increase if


1. Earth revolves faster around the 100. In a spacecraft, it is possible to
Sun. create "artificial gravity" by
2. You travel very fast on earth a) Surrounding the spacecraft by
3. You cross the troposphere and heavy water (deuterium)
reach stratosphere b) Shooting the spacecraft from the
Select the correct answer using the codes ground at a very high speed
below. c) Use material in the spacecraft that
can shield against the lack of
a) 1 and 2 only gravity in space
b) 2 and 3 only d) Spinning the spacecraft on its axis
c) 1 only
d) None of the above

http://www.insightsonindia.com INSIGHTS Page 19






SPACE FOR ROUGH WORK

http://www.insightsonindia.com INSIGHTS Page 20






SPACE FOR ROUGH WORK

http://www.insightsonindia.com INSIGHTS Page 21






SPACE FOR ROUGH WORK

http://www.insightsonindia.com INSIGHTS Page 22

You might also like